UCC Sales: Lawrence

UCC SALES OUTLINE

 

I. Scope of Article 2

·         Article 2 applies to present transactions in goods and contracts to sell goods at a future time. Courts must determine whether a transaction is governed by Article 2.

 

§2-103(j), Good Faith:  means honesty in fact and the observance of reasonable commercial standards of fair dealing.

 

A. What are goods? UCC 2-103(k) – goods means all things (including specially manufactured goods) which are movable at the time of identification to the contract for sale.

o   Goods must be both existing and identified before any interest in them may pass.  Goods that are not both existing and identified are future goods.  §2-105(1)

o   Article 2 applies to existing goods and future goods (goods that do not exist at the time the contract was made).

 

o   Identification: UCC 2-501(1) – The buyer obtains a special property and insurable interest in goods by identification. Identification occurs:

(a) when the contract is made if it is for the sale of goods already existing and identified;

(b) if the contract is for future goods other than those in (c), they will be considered identified when the goods are shipped, marked, or otherwise designated by the seller as the goods to which the contract refers. 

(c)  when the crops are planted or otherwise become growing crops (see statute for full def.)

 

§  Insurable interest – an interest in property which would allow the owner to insure it. (one can’t insure someone else’s property). 

 

o   Personal Property can be divided between goods (tangibles) and intangibles (ex: licenses, trademarks). The question is what are you buying? Are you buying the good or are you buying a person’s intellectual power or ideas? The law generally tries to look past the form of a transaction and focus on the functional reality.

o   There is a middle ground called Indispensable Paper which has attributes of both tangible and intangible. It must be on paper, so it is tangible, however the rights that the paper conveys is intangible. Negotiable Instruments (negotiable documents) – ex: checks, promissory notes.

o   But we will focus on goods.

 

o   Foster v. Colorado Radio Corp.

§  Plaintiff, Colorado, sues defendant, Foster, for Foster’s alleged breach of promise to purchase certain assets of a New Mexico Radio Station.

§  The transaction involved both tangible (furniture, equipment) and intangible (license) property.

§  The Trial Court concluded that the contract did not fall within the UCC Article 2 b/c it did not deal w/ goods. It looked at the contract as a whole, used the dominant purpose test, and ruled that the dominant purpose was the sale of the intangible goods (the license). The based their ruling on Epstein.

§  The Court of Appeals reversed in part and affirmed in part. The court looked at the contract in two pieces, one for the sale of goods and one for the sale of non-goods. The real property and the intangible property parts of the contract are not w/in Article 2. However, the furniture and equipment sales are governed by Article 2.

§  The Court of Appeals ruled that Epstein is inapplicable to this case b/c in that case, the beauty service and the sale of beauty products was inseparable and the main purpose was the service. But in this case, there was no hybrid goods-service. It was only the sale of goods and non-goods, so it can be separated so that some falls under Article 2 and some doesn’t.

§  Most courts follow the dominant purpose test where the whole transaction either falls under Article 2 or none of it does. No separation.

 

B. What is a transaction?


 

II. Offer and Acceptance

UCC § 2-204, Formation in General

·         (1) A K for the sale of goods may be made in any manner sufficient to show agreement, including conduct by both parties which recognizes the existence of such a K.

o   appropriate conduct by the parties may be sufficient to establish an agreement

·         (2) An agreement sufficient to constitute a K for sale may be found even though the moment of its making is undetermined.

o   Directed primarily to the situation where the interchanged correspondence does not disclose the exact point at which the deal was closed, but the actions of the parties indicate that a binding obligation has been undertaken

·         (3) Even if one or more terms are left open a K for sale does not fail for indefiniteness if the parties have intended to make a K and there is a reasonably certain basis for giving an appropriate remedy.

o   If the parties intend to enter into a binding agreement this subsection recognizes that the agreement as valid in law despite missing terms, if there is any reasonably certain basis for granting a remedy.

§  The more terms the parties leave open, the less likely it is that they have intended to conclude a binding agreement, but their actions may be frequently conclusive on the matters despite omissions.

·         (4) Amended to include Electronic Transactions

o   Courts have said that you don’t look at how one electronic agent worked with another electronic agent; you still look at the parties’ behavior

 

UCC § 2-206, Offer and Acceptance in Formation of Contract

·         (1) Unless otherwise ambiguously indicated by the language or circumstances

o   (a) an offer to make a K shall be construed as inviting acceptance in any manner and by any medium reasonable in the circumstances.

§  Any reasonable manner of acceptance is intended to be regarded as available unless the offeror has made quite clear that it will not be acceptable. 

§  This section is intended to remain flexible and its applicability to be enlarged as new media of communication develop or as the more time saving present day media come into general use.

o   (b) An order or other offer to buy goods for prompt or current shipment shall be construed as inviting acceptance either by a prompt promise to ship or by the prompt or current shipment of conforming or non conforming goods, but such a shipment of non conforming goods does not constitute an acceptance if the seller seasonably notifies the buyer that the shipment is offered only as an accommodation to the buyer.

§  Either shipment or prompt promise to ship is made a proper means of acceptance of an offer looking to current shipment.

§  Deals with the situation where a shipment made following an order is shown by a notification of shipment to be referable to that order but has a defect.

·         Such a non - conforming shipment is normally to be understood as intended to close the bargain, even though it proved to have been at the same time of the breach.

§  However, the seller by stating that the shipment is non conforming and is offered only as an accommodation to the buyer keeps the shipment or notification from operating as an acceptance.

§  Countering the unilateral K trick (book notes)

·         Under pre code law, S could ship non conforming goods in response to an offer and then wait to see what happened.

o    If B accepted goods, then K resulted: with S as offeror of non conforming goods and B is the offeree

o    IF B brought suit, S could avoid liability by denying the existence of a K b/c shipping non conforming goods was not an act of acceptance requested by the B.

·         Under this provision, shipment of nonconforming goods amounts to an acceptance (unless S notifies B that shipment is being made as an accommodation) of the buyers offer.

·         (2) Where the beginning of a requested performance is a reasonable mode of acceptance an offeror who is not notified of acceptance within a reasonable time may treat the offer as having lapsed before acceptance.

o   The beginning of performance by an offeree can be effective acceptance so as to bind the offeror only if followed within a reasonable time by notice to the offeror.

o   For the protection of both parties it is essential that notice follow in due course to constitute acceptance.

 

Book Notes:

§1-103(3)(b)  Unless displaced by a specific UCC provision, the C/L applies.

§  OFFER

·         The code defers to many of the common law rules regarding offers.

o   E.g. advertisements, circulars, and price quotas still are not viewed generally as offers.

o   Courts interpreting the code will continue to follow the Common law.  It’s revolutionary because it’s saying that it’s not the only source of law. 

·         Restatement of Judgments §24

o   An offer is the manifestation of willingness to enter into a bargain so made as to justify another person in understanding that his assent to his bargain is invited an will conclude it.

§  ACCEPTANCE

·         Code: unless the language or the circumstances make it clear that the offeror is insisting on an exclusive method of acceptance, the offeree can accept in any manner reasonable under the circumstances.

o   An offeree who chooses to accept by making a return promise can also do so by way any reasonable medium.

§  Most offerees do so by making a return promise or by beginning a requested performance/

o   Even though the offeree’s conduct in beginning performace is treated as a return promise, it does not have to be communicated to be effective.

§  Protects the offerees reasonable reliance, but creates problems for the offeror

o   §2-206(2)

§  where the beginning performance is a reasonable mode of acceptance an offeror who is not notified of acceptance within a reasonable time may treat the offer as having lapsed before acceptance.”

·         Code balances interests of the parties

o   E.g. Tu: seller receives order of specially manufacture equip

§  Seller begins work, but does not tell buyer and on Fri, buyer revokes.

§  On these facts, S’s conduct amounts to an acceptance and the buyers revocation is ineffective.

·         The only issue is whether the 3 day delay giving notice was unreasonable and cases to date give little guidance to what is a reasonable time.

 

 

§ 2-205, Firm Offers – An offer by a merchant to buy or sell goods in a signed writing which by its terms gives assurance that it will be held open is NOT revocable, for lack of consideration, during the time stated, and if no time is stated, for a reasonable time, but in no event may such period of irrevocability exceed three months.

·         This only applies to contracts for the sale of goods.

·         “signed” does not have to be a complete signature.

 

UCC § 2-104(1), “Merchant”a person who deals in goods of the kind, or by his occupation, holds himself out as having knowledge or skill peculiar to the practice or goods involved in the transaction.

 

§2-209- Modification, rescission and waiver

·         This section seeks to protect and make effective all necessary and desirable modification of sales K without regard to the technicalities which at present hamper such adjustments.

·         (1) an agreement modifying the K within this needs no consideration to be binding.

o   Official Comment 2 to §2-209: 

§  Even though don’t need consideration to be binding, modifications made thereunder must meet the test of good faith imposed by this act.

§  The test of good faith between merchants or as against merchants includes observance of reasonable commercial standards of fair dealing in the trade (§2-103), and may in some situations require an objectively demonstrable reason for seeking a modification.

·         Such matters as a market shift which makes performance come to involve a loss may provide such a reason even though there is no unforeseen difficulty as would make out a legal excuse from performance under 2-615, and 2-616.

·         (2) An agreement in a signed record which excludes modification or rescission except by a signed writing cannot be otherwise modified or rescinded, but except as between merchants such a requirement on a form supplied by the merchant must be separately signed by the other party.

o   Comment 3 to §2-209:

§  Permits the parties in effect to make their own SOF as regards any future modification of the K by giving affect the clause in a signed agreement which expressly requires any modification to be by signed writing.

·         But note that if a consumer is to be held to such a clause on a form supplied by a merchant it must be separately signed.

·         (3) The requirement of the statute of frauds section of this Article (§ 2-201) must be satisfied if the K as modified is within its provisions.

o   Comment 3 to §2-209

§  Subsections (2) and (3) are intended to protect against false allegations of oral modifications.

·         “modification or rescission” includes abandonment or other exchange by mutual consent.

·         It does not include unilateral “termination”(§2-106) and cancellation §2-106(4).

o   Statute of Frauds provisions are expressly applied to modifications by subsection (3).

§  Under these provisions the delivery and acceptance test is limited to the goods which have been accepted, that is, to the past.

§  “Modification” for the future cannot therefore be fabricated up by oral testimony if the price involved is $500,000 or more since the modification must be shown at least by an authenticated memo.

·         Since a memo is limited to its effect to the quantity of goods set forth in it there is safeguard against oral evidence.

·         (4) Although an attempt at modification or rescission does not satisfy the requirements of subsection (2) or (3) it can operate as a waiver.

o   Official Comment 4:

§  This section is intended to prevent contractual provisions excluding modification except by a signed writing rom limiting in other respects the legal effect of the parties actual later conduct. The effect of such conduct is further  regulated by subsection (5).

·         (5) A party who has made a waiver affecting an executory portion of the contract may retract the waiver by reasonable notification received by the other party that strict performance will be required of any term waived, unless the retraction would be unjust in view of a material change of position in reliance on the waiver.

 

 

Hill v. Gateway 2000, Inc.

·         Hill purchased a computer from Gateway over the telephone. Included in the box was an arbitration clause, which would govern any dispute if the customer did not return the computer within 30 days. Hill kept the computer longer than 30 days.

·         Hill argues that although they read the agreement, the arbitration clause did not stand out, and they did not read the agreement closely.

·         The court ruled that UCC §2-207 does not apply in a case b/c there is not a “battle of the forms”, but only one form. The question is not whether the terms were added, but when the contract was made.

·         If a contract was formed at the time of the phone call, then there was an offer (by Hill) and the acceptance (by Gateway). All of the terms in the agreement in the box are mere proposals to add terms to the contract. So UCC §2-207 would apply, and since Hill is not a merchant, the terms are not automatically added. But Judge Easterbrook rules that there was no contract formed by phone call. The contract was formed when Hill did not return the computer within 30 days. So the offer was made by Gateway, and the acceptance was made by Hill. Thus, no terms were added and UCC §2-207 is irrelevant.

Misc. Notes

·         It is not beneficial to require parties to adhere to a formalistic creation of a contract. Some indefinite terms should be allowed, and an indefinite term should not render a contract void. Thus, the basis for UCC §2-204.

·         During Hill’s telephone call w/ Gateway, the telephone operator did one of two things: 1) Promised to ship the computer or 2) shipped the computer. This seems to fall under UCC § 2-206, which created the contract at the moment of the telephone call. So why did the court rule that the contract was formed only when Hill didn’t send the computer back w/in 30 days?

·         Prof. Lawrence says that Judge Easterbrook arbitrarily decided that the contract was formed when Hill didn’t send the computer back. The problem w/ Easterbrook’s analysis is that if a contract was already formed at the time of the telephone call, then the terms of the form w/ the computer can only be considered as proposals of additional contract terms.

·         The Hill decision has not gone down well w/ academics b/c it ignores and destroys fundamental contract doctrine. It does not address the legal significance of the exchange between the parties during the telephone call, b/c it is a classic case of UCC § 2-206, which the court ignored.

·         Other jurisdictions have both supported and rejected this decision.


 

 

III. Statute of Frauds

      A. The Reasoning Behind the Statute of Frauds (2-201)

o   Even though some contracts are properly formed, there are some that will not be enforced unless they are reduced to writing.

o   This statute is designed to prevent fraudulent testimony about contracts.

o   The writing does not necessarily mean that there is a contract. It’s just that it is corroborative evidence other than oral testimony.

o   The SOF creates a preliminary procedural hurdle that the plaintiff must overcome b/f the case goes to the jury. If can’t get past the hurdle (or through one of the exceptions), then SOF bars enforcement of the contract.

 

B. § 2-201, Statute of Frauds

(1)   A contract for the sale of goods for the price of $500 or more is NOT enforceable by way of action or defense unless there is some writing sufficient to indicate that a contract for sale has been made between the parties and signed by the party against whom enforcement is sought, or by his authorized agent or broker. A writing is not insufficient b/c it omits or incorrectly states the term agreed upon but the contract is NOT enforceable under this paragraph beyond the quantity of goods shown in such writing.

·                     This does not require that all of the terms of the agreement be in writing. It does not even require that all express terms be in writing. All that is explicitly required for an effective writing is:

 

i) the signature of the party against whom enforcement is sought (or his authorized agent or broker);

§  Doesn’t have to be signature exactly. Can be symbol, letterhead, printed name, etc.

§  UETA (Uniform Electronic Transactions Act)

a) Electronic signatures are OK

b) Computerized contracting, where one person’s computer contracts w/ another. This can form a contract even though no one reviews the contract or its terms. One can’t use the defense that machines can’t make contracts, b/c the fact that you programmed the machine is your manifestation of assent.

§  E-Sign Act – provides that the documents are legally effective even though the electronic signature is not in paper format.

 

ii)  sufficient information to identify the parties and show that a contract for sale exists between them;

 

iii) a quantity term;

§  The quantity term requirement is not stated in the rule, but was implied by the courts.

§  A contract will not be enforced beyond the quantity shown in the writing.

§  The reason why quantity is so important (but not price) is that quantity varies more than price (which can be gap-filled by court). Also, quantity also affects the price (Costco style).

 

iv)  Description of the item (hereafter added by Prof. Lawrence) – since SOF only applies to sale of goods, a description will indicate that we are dealing w/ goods; AND

 

v) Must suggest that this is a sales transaction.

 

·                     The writing need not be contained in a single document. Several writings can be used to satisfy the Statute of Frauds. However, there is debate as to whether sufficient cross-reference is required between the separate writings.

·                     The production of sufficient writing does not of itself establish that there is an enforceable agreement. The defendant can still introduce evidence to show that, despite the writing, the parties never entered into a binding agreement.

 

(2)   This is a partial exception to the writing requirement of subsection (1) – Between merchants, a writing that would satisfy the SOF as to the merchant who signed it can also satisfy the SOF as to another merchant who has not signed it, if written notice of objection is not given w/in 10 days after the confirmation is received. The seven requirements under this section are:

1.                Both parties MUST be merchants (2-104, “merchants”);

2.                Confirmation memo must be sent w/in reasonable time;

3.                the memo has to confirm the contract;

4.                the memo must bind the sender (if it doesn’t bind the sender, then it shouldn’t bind the recipient);

5.                the memo must be received (mailbox rule does not apply, must be physically received by recipient);

6.                the party receiving the memo has reason to know of its contents; AND

7.                the other party does not object to its contents w/in 10 days of receipt of the confirmation.

·         This does not necessarily mean that a contract was created b/c a defendant can still introduce evidence that no contract existed or contained different terms. This only prevents a defendant from using the SOF defense.

·         This section wants to encourage the formation of contracts, especially over the phone. A contract that is made over the phone should have a confirmation sent by at least one of the parties.

·         If one party sends this confirmation and the confirmation will bind the sender under subsection (1), then the other party is bound as well, if they satisfy subsection (2).

·         HYPOS – suppose Jones and Ames have an oral contract for the sale of machines. Ames prepares a memorandum in confirmation. How do the following facts affect Ames’ ability to use UCC 2-201(2)?

a) Ames writes  a memo but doesn’t send it – Not received by other party, so Ames is screwed.

b) Ames faxes a copy to Jones after consulting w/ his lawyer – must look at whether it is a reasonable time. Is it reasonable?

c) Ames sent the memo w/in reasonable time, but it got lost in the mail – the party must physically receive it, so Ames is screwed. No mailbox rule.

d) If Jones does receive the memo w/in a reasonable time, but Jones doesn’t read it – this is a case that the recipient does not know of the memo’s contents. However, Jones had “reason to know of its contents”, meaning that he should have known. So, Ames is OK.

e) If a typo in the memo says that it is for 10 units, but the oral contract was actually for 100 units – If Jones sues Ames and Ames raises the SOF, then Jones can produce the writing to sue Ames, but can only enforce it for 10 units.

·         If Ames sues Jones, can also only enforce the contract for 10 units b/c the writing that is sent by Ames must be “sufficient against the sender.” Since Ames can only enforce 10 units against Jones, Jones can only enforce 10 units against Ames. Under section (1), it is not enforceable “beyond the quantity of goods shown in such writing.”

f) If Ames is a lawyer – then the contract would not be between merchants, so subsection (2) does not apply.

g) If Jones receives the memo, but then immediately objects to the terms of the memo – Jones would have to send a writing back to Ames. Picking up the phone is not sufficient b/c the sender also needs documentation of the objection.

h) The objection letter by Jones to Ames is sent w/in 10 days but was lost in the mail – Even though the confirmation letter is required to by “physically received”, the written notice of objection only needs to be “sent” (the code states “is given w/in 10 days). Thus, the objection by Jones is effective b/c it was “given” w/in 10 days.   

 

(3)   There are three full (no writing involved) statutory exceptions to the writing requirement:

(a)    if the goods are to be specially manufactured for the buyer and are not suitable for sale to others in the ordinary course of the seller’s business and the seller, b/f notice of repudiation is received and under circumstances which reasonably indicates that the goods are for the buyer, has made either a substantial beginning of their manufacture or commitments for their procurement; OR

·         All of these conditions must be satisfied. There are many pitfalls. Thus, not many situations will qualify under this exception.

·         The difficult provision of this exception is “under circumstances which reasonably indicate that the goods are for the buyer…” It is difficult to satisfy this provision.

·         An oral contract for the delivery of several units, that satisfies UCC 2-201(3)(a), is enforceable for the entire contract quantity, even though the seller has only begun manufacturing a part of the total quantity.

o    Prof. Lawrence does not agree w/ the above rule and thinks the enforcement should be limited only to the extent that the seller has committed to manufacturing. Furthermore, the above rule seems to go against the spirit of the UCC in regards to the importance of quantity.

 

(b)   if the party against whom enforcement is sought admits in his pleading, testimony or otherwise in court that a contract for sale was made, but the contract is not enforceable beyond the quantity of goods admitted; OR

·         An admission does not have to be a legal conclusion. Should pursue the facts which establish offer and acceptance between the parties, since most laypeople don’t understand whether a contract is formed. Thus, a layperson could deny it and not realize that a contract was formed.

 

(c)    With respect to goods for which payment has been made and accepted or which have been received and accepted.

·         Ex: Payment was made to the seller and the seller accepts it. The corroborative evidence of the existence of a contract is that the seller has accepted the payment.

·         Ex: Goods were received and accepted by the buyer. Upon receipt, the buyer has a choice to either refuse the goods or accept them. If the buyer accepts them, then this is corroborative evidence that a contract was made.

 

Lockwood v. Smigel

·                     The offer was an oral offer to sell a Rolls Royce. $100 was made as a down payment.

·                     The seller later refused to sell the car, invoking the SOF defense under UCC 2-201(1).

·                     How can the buyer enforce the contract?

o    The $100 down payment can constitute a payment received by the seller and made by the buyer under UCC 2-201(3)(c). Therefore, the SOF defense is eliminated from this UCC section.

§  The down payment was only a small piece of the purchase price, and court must determine whether the down payment can constitute a payment under the UCC.

o    The court focuses on the intent of the drafters and looks at the Official Comments of the UCC, “where the goods are apportionable, the contract is enforceable only as to the portion for which payment has been made.”

o    The court holds that this does constitute a payment because the car is an indivisible unit. We cannot only enforce the portion for which payment has been made b/c the car is one whole unit. Thus, the deposit constitutes payment and the SOF is eliminated.

o   The court wants to advance the policy of avoiding disputes over quantity. This is a major theme seen throughout the UCC Article 2.   

 

C. Non-Statutory Exceptions – Courts use the following two doctrines to overcome the SOF requirements and give relief to the plaintiff:

1) Equitable Estoppel – requires a showing by the plaintiff that the defendant made one of the following affirmative misrepresentations:

                        a) that the SOF had been satisfied;

b) that the D would prepare a signed writing in the future; OR

c) that the SOF would not be used as a defense.

 

2) Promissory Estoppel – often requires a great deal of reliance, sometimes amounting to “unconscionable injury.” Sometimes requires “definite and substantial” reliance.

 

            D. The Effects of Noncompliance

·         Noncompliance w/ the SOF generally renders a contract unenforceable. This is NOT the same as void. Unenforceability refers to the lack of a remedy.

·         Void means that the contract does not exist. Unenforceable contracts may become enforceable at a later time.

·         The SOF is an affirmative defense. It must be pleaded or it will be considered waived. The party seeking to overcome the SOF will have the burden of showing that the SOF has been satisfied.

 

Southwest Engineering Company v. Martin Tractor Company

·                     During a meeting between the representatives of the two companies, where Martin produced a memorandum that listed certain products, w/ prices, and quantity, for sale to Southwest. The writing listed the name and title of Martin’s representative.

·                     The defendant argued that this memorandum does not constitute a writing that is a contract for the sale of goods. This writing is only part of the negotiations and was only an offer. Therefore, the SOF applies, so no contract is enforceable. They are arguing for a very literal reading of UCC 2-201(1). But Prof. Lawrence states that UCC 2-201 should not be read that tightly and should be read liberally to allow more contracts to be enforced.

·                     The Kansas Supreme Court ruled that under UCC 2-201, there was an agreement reached during the meeting and that the memorandum was a contract for the sale of goods. Used a relaxed reading of UCC 2-201. There are only three requirements:

1) the writing must evidence a contract for the sale of goods (should state the goods and that the transaction is a sale);

2) it must be ‘signed’, a word which includes any authentication which identifies the party to be charged; AND

3) it must specify quantity.

·                     Prof. Lawrence states that the Court got this wrong b/c satisfaction of the SOF does not indicate that a contract has been made. This is a separate question. The only question for the SOF is whether the writing satisfies the SOF. If the plaintiff can satisfy the SOF, then can determine whether there was a contract or not. The SOF is narrow in its scope.

 

Harry Rubin & Sons v. Consolidated Pipe Co.

·                     Plaintiff sues for enforcement of three separate oral agreements. Plaintiff sent purchase orders as confirmation memorandum for the oral contracts, to the defendant.

·                     The issue is whether the confirmation memos satisfy the SOF under UCC 2-201(2).

·                     The Court rules that both of the memos satisfies the SOF so that both oral contracts are removed from the SOF.

 


 

CHAPTER 4: Sources of Contract Terms

I. Terms Supplied by the Express Agreement

·         This is the most obvious.

·         “Freedom of contract” is the general rule.

·         UCC § 1-302(a) – except as otherwise provided in subsection (b) or elsewhere in the UCC, the effect of provisions of the UCC may be varied by agreement.

o   UCC § 1-201(3), “Agreement” – the bargain of the parties in fact, as found in their language or inferred from other circumstances, including course of performance, course of dealing, or usage of trade.

§  “Contract” is different from “Agreement” in that it encompasses an agreement. Contract is not only an agreement, but a contract also creates legal rights in the parties.

 

II. The Parol Evidence Rule

·         The extent to which oral evidence of additional express terms to a contract will be excluded. The evidence may or may not exclude certain types of evidence.

·         The SOF is a procedural hurdle. It does not govern contract terms. The PER is a rule concerning admissibility of extrinsic evidence for the purpose of establishing an express term of the contract other than those listed in the contract.

·         The writing is potential evidence of a contract. Just b/c there is a writing doesn’t necessarily mean that there is a contract.

·         The main thing is a determination of the actual intent of the parties.

 

UCC § 2-202(1), Final Written Expression: Parol or Extrinsic Evidence

You cannot contradict an “integrated” (final terms) agreement with evidence of any prior agreement or of a contemporaneous oral agreement, but it may be explained or supplemented:

(a) by course of performance, course of dealing, or usage of trade(§1-303); and

(b) by evidence of consistent additional terms unless the court finds the writing to have been intended also as a complete and exclusive statement of the terms of the agreement.

·         Must determine whether it is partially (the terms are final, but the writing is not an exclusive statement of all of the terms) integrated or completely (the terms are final and the writing is an exclusive statement of all of the terms) integrated.

·         If either partially or completely integrated, can NOT introduce evidence of contradictory terms.

·         If partially integrated, can introduce evidence of additional terms.

·         If completely integrated, can NOT introduce evidence of additional terms.

·         The problem w/ the above rule is that if we have ample evidence of the intent of the parties, then the rule works easily. But if we don’t have evidence of the intent of the parties, then it is hard to determine whether the parties intended a partial or complete integration, or whether it was intended to be integrated at all.

·         The UCC approach deviates somewhat from the Common Law approach. The main difference is in the guidelines for construing intent.

o    Comment 3 – “If the additional terms are such that, if agreed upon, they would certainly have been included in the document in the view of the court, then evidence of their alleged making must be kept from the trier of fact.”

 

o    Contrast this w/ the Natural Inclusion Test of the common law. “Courts should keep the evidence out if the evidence is for a term that would have been naturally included in the contract.”

           

A. The Common Law

·         When parties to a contract have reduced their agreement to a writing which they intend to be a final expression of at least some of their understandings, they may not introduce evidence of prior oral or written terms, or contemporaneous oral terms, that contradict the writing. They may, however, attempt to supplement the writing by introducing evidence of consistent additional terms. If they intend that the writing be both final and complete, even evidence of consistent additional terms will be excluded.

·         Steps in determining whether to invoke the Parol Evidence Rule:

1) Ascertain whether the parties have adopted a particular writing as a final expression of at least part of their agreement.

·         A writing is final when the parties have gone beyond the negotiating stage and have agreed that the language represents at least some of their understandings w/ respect to a contract.

 

2) Determine whether the writing is complete.

·         A writing is complete when the parties intend for it to represent all of their understandings w/ respect to the contract.

·         Four Corners Doctrine – early common law approach to determine whether complete. Two steps:

a) The judge would first look at the writing to see whether it appeared complete;

b) If the judge concluded that it did, the next step was to presume that it was complete. Neither party could convince the court that what they actually intended was a partial integration.

·         However, a party was still free to show that the agreement represented by the writing was not the only contract between the parties.

·         Collateral Contract – evidence of a collateral agreement, not supported by separate consideration, might be admissible to supplement the terms of the apparently complete writing, if the term was one that a reasonable person “would not ordinarily be expected to embody in the writing.”

 

III. Usage of Trade, Course of Dealing & Course of Performance

·         All of these can provide a source of terms to the contract which are just as much a part of the contract as the express terms.

 

·         UCC § 1-103 – (a)The UCC must be liberally construed and applied to promote its underlying purposes and policies, which are:

(a)(2) to permit the continued expansion of commercial practices through custom, usage, and agreement of the parties;

 

UCC § 1-303, Course of Performance, Course of Dealing, and Usage of Trade

(a)   Course of Performance – a sequence of conduct between the parties to a particular transactions (installment contract) that exists if:

1) the agreement of the parties w/ respect to the transaction involves repeated occasions for performance by a party; AND

2) the other party, w/ knowledge of the nature of the performance and opportunity for objection to it, accepts the performance or acquiesces in it w/o objection.

(b) Course of Dealing a sequence of conduct concerning previous transactions (prior contracts) between the parties to a particular transaction that is fairly to be regarded as establishing a common basis of understanding for interpreting their expressions and other conduct.

·         This is essentially the evolution of a private usage of trade between the parties.

·         “fairly” – the transactions must be as comparable as possible. A transaction for 1000 units cannot be expected to be performed in the same manner as a transaction for 10 units.

 

(c) Usage of Trade – any practice or method of dealing having such regularity of observance in a place, vocation, or trade as to justify an expectation that it will be observed w/ respect to the transaction in question.

·         “place” is an interesting issue b/c there are regional differences in terms of custom.

(d) COP, COD, of UT can be used for interpretation or establishing terms of a contract.

(e) The express terms of an agreement and any applicable COP, COD, or UT must be construed whenever reasonable as consistent w/ each other. If such a construction is unreasonable, then the hierarchy, from most controlling to least:

1) Express Terms 2) COP 3) COD 4) UT

 

Hypo Problem

·         Seller & buyer have a contract requiring “prompt shipment”. Seller delivers the goods after 52 days. Buyer sues for breach.

 

·         prompt shipment

o   What type of UT would help make a case for the seller?

§  If the business backdrop of UT is that “prompt” means shipment w/in 60 days, then shipment after 52 days may not necessarily breach the contract.

o   What type of COD would help make a case for the seller?

§  If this is their first contract together, then there is no COD.

§  If there was prior contracts between the parties, which of the contracts is relevant to this contract? How did the parties act in the previous contracts in regards to “prompt shipment”?

o   What type of COP would help make a case for the seller?

§  There must be some sort of installment or ongoing contract between the two parties. Must have a sequence of performance between the parties under the existing contract.

·         However, what constitutes a “sequence” is not defined. Probably two prior installments is the minimum. There must be enough of a sequence to create a legitimate expectation between the parties.

§  Must also show that several of those prior shipments were made around the 52 day mark and the buyer never objected. 

 

Relationship between UCC § 2-202(a) and UCC § 1-303(e)

·         Some courts use to automatically hold that under UCC § 2-202, if COP, COD, or UT contradict the terms of the contract, then they are inadmissible.

·         However, under UCC § 1-303(e), the express terms, COD, COP, and UT should be construed as consistent w/ each other, if it is reasonable to do so.

·         Prof. Lawrence & the Comments to UCC § 2-202 argues that courts should rule that if there are contradictory terms, it is a question of fact to be resolved by a jury. The jury must determine what the parties intended. The court ought to be willing to listen to any evidence that the parties offer to show what their intent was.

o   The only time that COP, COD, and TU should be excluded when dealing w/ contradictory terms under UCC § 2-202(a), is when the express term was intended to be a complete and exclusive statement of the term (the contract specifically excludes COD, COP, and UT).

·         Notice that under UCC § 2-202, even if the express terms are intended to be completely integrated, then can still present evidence of UT, COD, and COP, although evidence of consistent additional terms is excluded under UCC § 2-202(b).

·         However, this hierarchy cannot be construed literally. Express terms and Trade Usage are actually both equally legitimate sources of contract terms.


 

IV. Implied in Law Terms (Gap-Fillers)

·         These gap-fillers are available to help prevent the contract from failing for indefiniteness.

·         We will also focus on the seller’s delivery obligations as well as the gap-filling provisions.

 

A. Open and Deferred Price Terms

o   A contract does not fail for indefiniteness for lack of specified price (unless the goods are so unusual that a price cannot be measured). The Code will supply a price in the contract.

 

o   § 2-305, Open Price Terms

(1)   The parties, if they so intend can conclude a contract for sale even though the price is not settled. In such a case, the price is a reasonable price (usually the market price) at the time of delivery if:

            a) nothing is said as to price; or

b) the price is left to be agreed by the parties and they fail to agree;

c) price is to be fixed in terms of some agreed market or other standard as set or recorded by 3rd party

 

(2)   A price to be fixed by the seller or by the buyer means a price for him to fix in good faith.

 

(3)   If a price left to be fixed otherwise than by agreement of the parties fails to be fixed through fault of one party, the other may at his option treat the K as cancelled or himself fix a reasonable price.

 

(4)   When, however, the parties intend not to be bound unless the price be fixed or agreed and it is not fixed or agreed there is no K. 

 

 

      B. Delivery and Payment Terms Generally

o   § 2-301, General Obligations of PartiesThe obligation of the seller is to transfer and deliver and that of the buyer is to accept and pay in accordance w/ the contract.

 

1) Place of Delivery

§  § 2-308, Absence of Specified Place of Delivery Unless otherwise agreed,

a) the place for delivery of goods is the seller’s place of business or if he has none, his residence; but

b) in a contract for the sale of identified goods which to the knowledge of the parties at the time of contracting are in some other place, that place is the place for their delivery;

·         Hypo: assume that the good is located in Bakersfield, and the parties enter into a contract to sell/purchase the good but no place of delivery has been specified. If both parties know that the good is in Bakersfield, then that becomes the place of delivery.

·         Comment 1: where delivery by carrier is “required or authorized by the agreement (preamble to 2-504”, the seller’s duties as to delivery of the goods are governed not by this section but by section UCC 2-504.

o    Most courts hold that if there is significant distance between the parties, it is implicit that the seller is authorized to send the goods by carrier. T

·         This Code section is in the context of an absence of an express place of delivery term. “No F.O.B. provision”. This is a gap-filler provision.

 

2) Time for Performance

§  § 2-309, Absence of Specific Time Provisions

(1)   The time for shipment or delivery or any other action under a contract if not provided in this Article or agreed upon shall be a reasonable time.

·         § 1-205“Reasonableness Time” Definition

1.      whether a time for taking action required (the UCC) is reasonable depends on the nature, purpose, and circumstances of the action.

·         Agreement as to a definite time however may be found in a term implied from contractual circumstances, usage of trade, or course of dealing or performance as well as in an express term.

a.       Such cases fall outside the subsection since in them the time for action is ”agreed” by usage.

 

(2)   If the K provides for successive performances but is indefinite in duration it is valid for a reasonable time unless otherwise agreed may be terminated at any time by either party.

                                                                                                                                       i.      The applicable principles make it clear that surprise is to be avoided good faith judgment is to be protected  and notice or negotiation to reduce the uncertainty to certainty is to be favored.

                                                                                                                                     ii.      The reasonable time of duration appropriate to a given arrangement is limited by the circumstances.

1.       When the arrangement has been carried on by the parties over years and years, the “reasonable time” can continue indefinetly and the K will not terminate until notice.

 

(3)   Termination of a K by one party except on the happening of an agreed event requires that reasonable notification be received by the other party and an agreement dispensing with notification is invalid if its operation would be unconscionable.

                                                                                                                     i.      This subsection recognizes that application of principles of good faith and sound commercial practice normally call for such notification of termination of a going K relationship as will give the other party a reasonable time to seek a substitute agreement.

1.       This is valid under this subsection unless the results in putting it into operation would be lead to an unconscionable state of affairs.

 

·         Hypo – Let’s assume the parties enter a contract and don’t specify anything about the time of delivery and there is no UT, COP, or COD. Thus, we must use the gap-filler.

o    B/f there is a breach of the contract due to passage of time, there must be a notification to the other party. The requirement of “good faith” requires reasonable notification b/f a contract may be treated as breached b/c a reasonable time for delivery or demand has expired.

o    A failure to reply to such a notification, indicates acquiescence and that the proposed time by the notifying party becomes a reasonable time.

o    If either party insists on a time for delivery, and the response indicates that this time cannot be met, then this is a “flat breach.”

 

o    UCC 2-609 – “can always make a demand for adequate assurance of performances.” A failure to provide adequate assurances will allow the case to move into repudiation.

 

     3) Shipment and Destination Contracts

§ 2-319, F.O.B. (Free on Board) and F.A.S. (Free Alongside) Terms

(1)   F.O.B. is an express delivery term (this is NOT a gap-filler provision). If there is no express F.O.B. provision, then 2-319 is irrelevant.

Ex: F.O.B. San Diego, $15,000 – Shipment Contract

·         Ex: F.O.B. New York $15,000 – Destination Contract

·         2-319(1)(a) is not the exclusive mechanism for creating a shipment contract. Can create a shipment contract through trade usage, course of dealing, or course of performance.

b.       (1) unless otherwise agreed the term FOB at a named place, even though used in only connection with the stated price is a delivery term

                                                                                                                                       i.      totally irrelevant unless have an expressed delivery agreement

                                                                                                                                     ii.      eg

1.       FOB SD $15,000: shipment K

2.       FOB NY $15,000: destination K

 

(a)   when the term is FOB the place of shipment, the seller must at that place ship the goods in the manner provided in this article § 2-504 and bear the expense and risk of putting them into the possession of the carrier.

o   Determines risk of loss: seller bears the risk of putting them into the carrier: after that, risk of loss goes to buyer.

o   Therefore, buyer needs to insure his risk once carrier obtains goods. 

o   2-319 is not exclusive mechanism for shipment K

§  could have one through other express terms: trade usage, COD, COP.

 

(b)   when the term F.O.B the place of destination the seller must at his own expense and risk transport the goods to that place and there tender delivery of them in the manner provided in this Article (§2-503);

 

(c)    When under either (a) or (b) the term is also F.O.B vessel car or other vehicle the seller must in addition at his own expense and risk load the goods on board.

a.       The term is F.O.B vessel the buyer must name the vessel and in an appropriate case the seller must comply with the provisions of this Article on the for of bill of lading. 

 

§  § 2-503, Manner of Seller’s Tender of Delivery

(1) Tender of delivery requires that the seller put and hold conforming goods at the buyer’s disposition and give the buyer any notification reasonably necessary to enable him to take delivery. The manner, time and place for tender are determined by the agreement and this Article, and in particular:

(a) tender must be at a reasonable hour, and if it is of goods they must be kept available for the period reasonably necessary to enable the buyer to take possession; BUT

(b) unless otherwise agreed the buyer must furnish facilities reasonably suited to the receipt of the goods.

(2) Where there is a “shipment contract”, tender requires that the seller comply w/ the provisions of UCC § 2-504.    

 

§  § 2-504, Shipment by Seller (Shipment Contracts) where the seller is required or authorized to send the goods to the buyer and the contract does NOT require him to deliver them at a particular destination (note the difference between “send” and “deliver”), then unless otherwise agreed he must:

(a)   put the goods in the possession of such a carrier and make such a contract for their transportation as may be reasonable having regard to the nature of the goods and other circumstances of the case; AND

(b)   obtain and promptly deliver or tender in due form any document necessary to enable the buyer to obtain possession of the goods or otherwise required by the agreement or by usage of trade; AND

 

(c)    promptly notify the buyer of the shipment.

 

§  Shipment Contract (see above, UCC § 2-504, replacing UCC § 2-319(1)(a)) – one in which the seller is required or authorized to send the goods to the buyer but the contract does not require him to deliver them at a particular destination. Ex: “F.O.B. seller’s city.” The seller’s responsibilities are:

1) put the goods in the possession of such a carrier, bear the expense and risk of putting then into the possession of the carrier (however, the buyer pays for the actual cost of transportation) and make such a contract for their transportation as may be reasonable having regard to the nature of the goods and other circumstances of the case;

 

2) obtain and promptly deliver or tender in due form any document necessary to enable the buyer to obtain possession of the goods or otherwise required by the agreement or by usage of trade; AND

 

3) promptly notify the buyer of the shipment.

§  Classification of shipment or destination contract is important b/c if shipment, seller only has to satisfy the above three requirements. After doing so, the risk of loss shifts to the buyer and shipment is proper. Any damage during transit will be the buyer’s responsibility to pursue recovery against the carrier.

 

 

§  Destination Contract – where the seller is required to deliver the goods at a particular location. Ex: “F.O.B. buyer’s city”

·         UCC § 2-319(1)(b) – when the term is F.O.B. the place of destination (destination contract), the seller must at his own expense and risk transport the goods to that place and there tender delivery of them in the manner provided in this Article.

·         UCC § 2-503(3) – Where the seller is required to deliver at a particular destination (destination contract), tender requires that he comply w/ subsection (1) and also in any appropriate case tender documents as described in subsections (4) and (5) of this section.

·         UCC § 2-503(1) - The seller’s responsibilities under a destination contract do not end until the seller is able to “put and hold conforming goods at the buyer’s disposition and give the buyer any notification reasonably necessary to enable him to take delivery.”

·         (a)  reasonable hour, and for a length of time reasonably necessary for B to take possession

·         (b)  unless otherwise agreed, B must furnish facilities reasonably suited to the receipt of the goods.

·         The risk of loss does not shift until the goods are tendered to the buyer at the designated destination as satisfying UCC § 2-503(1).

·         UCC § 2-504 does not apply here, b/c that is for shipment contracts.

 

§  HYPO: If there is no F.O.B. provision, no express terms in regards to delivery and no Trade Usage, no COP, no COD, how are the goods to be delivered?

·         UCC § 2-503, Comment 5 states that the gap-filler is the “shipment contract”. A “destination contract” must be specifically bargained for. Otherwise, we assume that there is a shipment contract.

 

    4) Payment Terms

§  UCC § 2-310, Open Time for Payment or Running Credit – Unless otherwise agreed:

(a) payment is due at the time and place at which the buyer is to receive the goods even though the place of shipment is the place of delivery;

·         If the seller wants earlier payment, it must be bargained for. UCC 2-310 is the default rule in the absence of an agreement that states otherwise. Must get it through agreement with the buyer either through express terms relevant TU CD CP.

·         “even though the place of shipment is the place of delivery” means that if there is a shipment contract, delivery occurs when the goods are turned over to the carrier. Thus, the buyer does not have to pay at that point. Payment is only due after the Buyer has received the goods, so the buyer can inspect the goods before accepting them. However, must still the consider the risk of loss shift to the buyer (???)

§  UCC § 2-511 – Tender of payment

§  UCC § 2-513Buyer’s right to inspect

 

        C. Open Quantity Terms (Output and Requirements Contracts)

§  Requirements Contract – where the buyer agrees to purchase all of his or her requirements for particular goods from the seller.

§  Output Contract – where the buyer agrees to purchase all of the seller’s output of particular goods.

 

§  UCC § 2-306, Output, Requirements and Exclusive Dealings

(1)   A term which measures the quantity by the output of the seller or the requirements of the buyer means such actual output or requirements as may occur in good faith, except that no quantity unreasonably disproportionate to any stated estimate or in the absence of a stated estimate to any normal or otherwise comparable prior output or requirements may be tendered or demanded.

 

(2)   a lawful agreement by either the S or the B for exclusive dealing in the kind of goods concerned imposes unless otherwise agreed an obligation by the S to use best efforts to supply the goods and by the buyer to use best efforts to promote their sale.


 

V. Supereminent Contract Terms

·         UCC § 1-302, Variation by Agreement

(a) Except as otherwise provided in subsection (b), or elsewhere in the UCC, the effect of provisions of the UCC may be varied by agreement.

(b) The obligation of good faith, diligence, reasonableness, and care prescribed by the UCC may NOT be disclaimed by agreement.

 

            A. Unconscionability

·         UCC § 2-302, Unconscionable Contract or Clause

(1)   If the court as a matter of law (exclusive determination by the judge) finds the contract or any clause of the contract to have been unconscionable at the time it was made the court may:

            i) refuse to enforce the contract; OR

ii) enforce the remainder of the contract w/o the unconscionable clause; OR

iii) so limit the application of any unconscionable clause as to avoid any unconscionable result.

§  Comment 1: This section is intended to allow the court to pass directly on the unconscionability of the K or a particular term of the K and to make a conclusion of law as to its unconscionability.  Courts are particularly vigilant against boilerplate language.  

 

§  Comment 1 (also): “The basic test is whether in the light of the general commercial background and the commercial needs of the particular trade or case, the clauses are so one-sided as to be unconscionable under the circumstances existing at the time of the making of the contract.”

 

§  The main concern of the drafters was to prevent oppression and unfair surprise.

§  Designed to help the poor, uneducated, and disadvantaged b/c they are the most frequent victims of sketchy practices.

 

§  Courts have not been very receptive to claims of unconscionability brought forth by merchants. Courts for the most part allow a free market economy. Courts are not going to retroactively save business people’s bad judgment.

 

§  In order to make a successful unconscionability claim, you must show BOTH:

a) Procedural Unconscionability – bad bargaining process.

·         Ex: a person doesn’t speak English. Lack of education makes it difficult to understand clauses. Fine print on back of contract forms. Fraud. High pressure sales tactics.

b) Substantive Unconscionability – overly harsh contract terms.

·         Ex: excessive price, undue restrictions on the debtor’s rights, or undue expansion of creditor’s remedies. Unreasonable liquidated damages provisions.

§  Procedural and substantive are weighed together on a sliding scale. The more of one that we find, the less of the other that we have to find in order to determine that there is unconscionability.  

 

(3)   When it is claimed or appears to the court that the contract or any clause thereof may be unconscionable the parties shall be afforded a reasonable opportunity to present evidence as to its commercial setting, purpose, and effect to aid the court in making the determination [of unconscionability].

 

b. Good faith

o   Can be viewed as an implied law: promise that cannot be excluded by parties: exception of rule of freedom of K

§  Limitations:

·         Implies only to performance and enforcement of a K

 

o   §1-304 Restatement: excluded negotiating good faith agreements

§  whenever we enter into a K, the duty arises. Duty arises out of the K itself.

§  Cumbersome to police these methods

§  Any K for sale of goods gives rise to a non excludible obligation to act in good faith

 

o   §1-201(20): defines good faith

§  Good faith in the case of a merchant means: “Honesty in fact”: extremely narrow range of applicability; AND the observance of reasonable commercial standards of fair dealing and trade.

·         Overall reasonable standard of fair dealing (unlike article 2, which is only from the particular trade).

·         Not just limited to merchants or merchants trade just like article 2.   

o   e.g.: dishonesty: cheating

o   doesn’t apply to negligence or open abuse of power, acting capriciously

 

o   §2-103(1)(j): in case of merchant good faith means honesty in fact and observance of fair dealing in the trade

§  Reserved in our book; no longer needed there if the jurisdiction has adopted §1-303

§  Correction in article 1: correct one of biggest drafting errors

·         before good faith defined as : honesty in fact (purely subjective standard)

o   Article 1 definition is superior: not limited to merchants

·         Definition: good faith in the case of a merchant means honesty in fact and reasonable standards of fair dealing in the trade. 

 

o   §1-303 Obligation of Good Faith – Every K or duty w/in the UCC imposes an obligation of good faith in its performance and enforcement.


 

 

IN ABSENCE OF A BREACH

§ 2-509 Risk of Loss in the Absence of a Breach

(1) If the K requires or authorizes the seller to ship the goods by carrier

(a) if it does not require him to deliver them at a particular destination, the risk of loss passes to the buyer when the goods are duly delivered to the carrier even though the shipment is under reservation (shipment K)

(b) if it does require him to deliver them at a particular destination and the goods are there duly tendered while in the possession of the carrier, the risk of loss passes to the buyer when the goods are there duly so tendered as to enable the buyer to take delivery. (destination K)

o    2-509(1) – only applicable when using third party common carrier, not when using seller’s own trucks

o    2-509(1)(a) shipment

o    2-509(1)(b) destination K

 

(2) Where the goods are held by a bailee to be delivered without being moved, the risk of loss passes to the buyer

(a)   on his receipt of a negotiable document of title covering the goods; OR

o    Once the buyer purchases the goods by purchasing a negotiable document to the goods, the title and risk of loss passes to the buyer and the seller no longer has any control over the goods.

§  The bailee is under strict liability to deliver (w/o moving the goods) the goods to the buyer.

o    The negotiable document serves three purposes:

1) serves as a receipt that the goods have been placed in the possession of the bailee;

2) serves as a storage contract; and

3) can pass title to the goods and shifts the risk of loss to the buyer

 

(b) On acknowledgement by the bailee of the buyer’s right to possession of the goods, OR

§  This subsection covers the case when the bailee does not issue any document at all.

§  The risk of loss does not pass until the bailee acknowledges the buyer’s right to possession.

§  Since the bailee is responsible for the goods, he’s just not going to give them over to anyone.

 

(c)  After his receipt of a non negotiable document of title or other written direction to deliver, as provided by subsection (4)(b) of §2-503.

§  This deals w/ a non-negotiable document. It serves the same first two purposes as a negotiable document (#1 & #2, above). However, title is not connected to the non-negotiable document. The document is only a receipt.

§  Title and risk of loss does not pass until the buyer takes the non-negotiable document to the bailee and asks for the goods. If the bailee complies, then title and risk of loss is passed.

 

(3) in any case not within subsection (1) or (2), the risk of loss passes to the buyer on his receipt of goods if the seller is a merchant; otherwise the risk passes to the buyer on tender of delivery.

o    “Receipt” is a term of art that means “actual physical possession.” UCC § 2-103(1)(c).

o    When the seller is a merchant, the merchant is the party to most likely have insurance. So the merchant should bear the risk of loss until actual physical possession of the buyer.

 

(4) The provisions of this section are subject to contrary agreement of the parties and to the provisions of this article on sale on approval (§2-327) and Effect of Breach on Risk of Loss (§2-510).

 

Eberhard Manufacturing Co. v. Brown

·         Facts:

o    Plaintiff, seller, brought action to recover for the price of goods sold and delivered to defendant, buyer, pursuant to a distributorship agreement. Defendant counterclaimed.

o    The trial court awarded $6,315.82 to the defendant on his counterclaim, $559.03 of which was a credit for goods lost in transit. The Circuit court affirmed.

o    The plaintiff argued that the court erred in giving the $559.03 credit b/c the risk of loss had passed to the defendant (buyer) when the goods were put on board the carrier. Argues that they had a “shipment contract”.

o    The defendant argued that they had a “destination contract” b/c the contract stated the words “ship to.”

·         Holding

o    The Court held that the contract contained no express term allocating the risk of loss. Thus, under UCC 2-503, Comment 5, if the contract does not have an express term regarding the risk of loss, then it is presumed as a “shipment contract.”

o    The fact that the contract contained the words “ship to” does not matter b/c the words “ship to” is included in all contracts where a carriage is contemplated.

·         Since the presumption of a shipment contract controls in this case, the trial court erred in awarding the $559.03 credit for the lost shipment.

 

Consolidated Bottling Co. v. Jaco Equipment Corp.

·         Consolidated entered into a contract where they would sell to Jaco, a used can filling machine. The contract contained the term F.O.B. purchaser’s truck.

·         Consolidated informed Jaco that if the Jaco could not take delivery of the machine w/in two weeks (around mid to late June), then Consolidated would prefer that delivery take place after July 4th. Jaco accepted.

·         On June 9, vandals broke into the warehouse and damaged the machine.

·         Plaintiff (seller), Consolidated, brought suit against defendant (buyer) Jaco, to recovery from Jaco the purchase price of $11,500 for certain used can filling equipment which was damaged b/f delivery was made.

·         Consolidated argues that the “risk of loss” had passed to Jaco and that it was holding the goods for Jaco, under UCC 2-503(1).

·         Jaco argued that since the contract provided no definite date of delivery, that a reasonable time had not passed (2-309), and therefore, the risk of loss remained w/ the seller, Consolidated.

·         The trial judge awarded judgment in favor of Jaco.

·         The Court of Appeals affirmed, holding that even if Consolidated had complied w/ UCC 2-503(1), it does not matter b/c the parties had provided their own express term regarding risk of loss, “F.O.B. purchaser’s truck.”

o    The risk of loss does not pass until the goods have been delivered to the buyer or have reached the place agreed upon.

o    Under UCC 2-319(1)(c), a term such as “F.O.B. purchaser’s truck” requires that the seller bear the expense and risk of loading the goods on board.

o    Even if the court stated that UCC 2-503(1) applied, which it does not b/c Consolidated is a merchant under UCC 2-509(3), then Consolidated would still lose b/c they still did not properly tender delivery.

 

Caudle v. Sherrard Motor Co.

·         Plaintiff (seller), Sherrard, entered into a contract w/ defendant (buyer), Caudle, where Caudle was to purchase a house trailer from Sherrard.

·         However, the trailer was not ready, and since Caudle had to go out of town and could not take possession of the trailer, Caudle told Sherrard that he would return later to get it.

·         B/f Caudle returned, the house trailer was stolen and Caudle stopped payment on the house trailer.

·         Sherrard brought suit against Caudle for sale price of the house trailer. The trial court found in favor of Sherrard, awarding him $6,285.70. Caudle appeals.

·         The issue is whether the risk of loss of a house trailer had passed from the seller, Sherrard, to the buyer, Caudle, b/f the house trailer was stolen from the seller’s premises.

·         Plaintiff argues that he was acting as a bailee of the trailer, and that by executing the contract, had acknowledged the D’s right to possession of the trailer. Under UCC 2-509(2), the risk of loss shifted to the buyer.

o    Plaintiff also argues that if he is not a bailee, then the risk of loss passed to the buyer under 2-509(4), where the parties had entered into a contract that is contrary to the rest of 2-509. A clause in the contract stated that “…no loss or damage or destruction of said motor vehicle shall release the buyer from his obligation hereunder.”

·         The Texas Court of Civil Appeals reversed, holding that the contract failed for want of consideration since the trailer was stolen b/f the risk of loss had passed to Caudle. Caudle had not taken actual physical possession of the goods and thus, the risk of loss did not shift.

o    The plaintiff is not a bailee. Under the Code, a bailee must be one that is “in the business of storing goods for hire – a commercial bailee.” In this case, the plaintiff is not in such a business, and thus, is not a bailee under 2-509(2).

·         The only time UCC 2-509(2) can apply is if there is an independent third-party bailee.

o    Further, the plaintiff’s argument about the contract clause (“no loss or damage…”) does not constitute a contrary agreement between the parties pursuant to UCC 2-509(4) b/c it should be interpreted as only protecting the seller after the buyer had taken possession of the trailer.

·         To interpret it otherwise, would be to impose on the buyer a difficult responsibility of insuring the trailer when they don’t even have possession of it.

 

Martin v. Melland’s, Inc.   

·         Buyer purchases a new truck and haystack mover while trading in his old equipment. While the seller was preparing the new truck and mover, the buyer held onto the old units, but title to the old units had already passed to the seller. The old units were destroyed while in the buyer’s possession b/f getting the new units.

·         There was no agreement concerning risk of loss. The seller refused to give the buyer the new units and the buyer brought suit.

·         The buyer argued that the correct standard is UCC § 2-509(2), where the buyer is a bailee. The seller had the right to take possession at any time, and thus, the risk of loss had passed to the seller.

o    The court struck down this argument b/c the buyer was not a bailee since he is not independent (3rd party) and there was no contract for bailment.

·         The ND Supreme Court holds that the proper standard is under UCC § 2-509(3), in that risk of loss will not pass until tender of delivery. Since the new unit was not to be delivered until it was ready, then delivery could not be tendered until the new unit was ready. Since the buyer is using the old units, he also has not tendered delivery of the old units to the seller. Thus, the risk of loss remained w/ the buyer and thus must bear the loss.


 

VII. The Effect of Breach on Risk of Loss

 

§ 2-510, Effect of Breach on Risk of Loss

(1)   Where a tender or delivery of goods so fails to conform to the contract as to give a right of rejection, the risk of their loss remains on the seller until cure or acceptance.

·         “Acceptance” means receipt and payment of the goods. As opposed to “reject”.

·         “Cure” - sellers makes changes to goods already tendered, such as repair, partial substitution, etc.  Comment 2.

o    Risk of loss does not pass until “cure” is complete. See Comment 2.

·         Perfect Tender Rule – the buyer has the right of rejection if goods do not conform in any way.

·         This section changes the rules of loss allocation that would otherwise apply in UCC § 2-509, to leave the risk of loss on the seller, when the buyer has the right of rejecting the goods (they are non-conforming).

·         Under this subsection, we are not concerned whether the buyer actually rejects or not. Only that the buyer has the right of rejection. If the goods are not conforming, the risk of loss does NOT shift to the buyer (as it would under UCC 2-509), but stays w/ the seller.

·         Whether the goods were conforming when shipped is a question of fact for the jury.

·         Under a shipment contract, if the seller ships conforming goods, then risk of loss passes to the buyer (2-509). If the seller ships non-conforming goods, then risk of loss stays w/ the seller (2-510).

 

(2)   Where the buyer rightfully revokes acceptance he may to the extent of any deficiency in his effective insurance coverage treat the risk of loss as having rested on the seller from the beginning.

·         The only time when this subsection applies is when the risk of loss has already passed to the buyer through “acceptance” under subsection (1). If the buyer then revokes his acceptance, subsection (2) applies.

·         Ex: The buyer accepts the goods, then discovers the extent of the defect, has the right to revoke, and properly does so. In the meantime, the goods are destroyed and the buyer has no insurance coverage (total deficiency in insurance coverage). The total risk of loss has passed b/c the deficiency is total.

o    If the buyer had 100% insurance coverage, then no risk of loss passes b/c the buyer’s insurance company covers the loss. The Code wants to pass the cost to the insurance company.

o    If the insurance covers only 1/3 of the loss, then the insurance company will pay 1/3 of the loss and the risk of loss is passed back to the seller w/ respect to the uninsured 2/3 of the loss.

o    Anti-subrogation provision. 

(3)   Where the buyer as to conforming goods already identified to the contract for sale repudiates or is otherwise in breach b/f risk of their loss has passed to him, the seller may to the extent of any deficiency in his effective insurance coverage treat the risk of loss as resting on the buyer for a commercially reasonable time.

·         This subsection deals w/ when the buyer breaches and how it effects the risk of loss.

·         This is also an anti-subrogation clause. Operates the same as subsection (2), except that subsection (2) is for the seller’s breach while subsection (3) is for the buyer’s breach.

·         There are several requirements that must be satisfied by the seller in order to pass the risk of loss:

1) Goods that the seller is producing must be conforming;

2) Goods are already identified to the contract b/f the buyer breaches; AND

3) Repudiation or breach by the buyer b/f risk of loss has already passed to the buyer.

4) Risk of loss rested on the buyer for a commercially reasonable time.

 

Multiplastics, Inc. v. Arch Industries, Inc.

·         Contract for the special manufacture of 40,000 lbs. plastic pellets which were to be delivered 1000 lbs. per day until all 40,000 lbs are delivered.

·         The seller made the goods and tried to contact the buyer to deliver them but the buyer kept ignoring the seller’s request. Finally, the buyer said that he would take delivery, but then continued to fail to issue a release.

·         While the seller was holding the goods, the seller’s plant burned down and the goods were destroyed.

·         The seller sued the buyer for the contract price.

·         B/c this case deals w/ a breach by the buyer, it is a UCC § 2-510(3) case.

o    The buyer breached b/c he had an obligation to accept the goods and to pay for them after the seller has properly tendered. The buyer did neither.

o    There is no question that the first three requirements of UCC § 2-510(3) are satisfied.

1) The goods were conforming

2) the goods were identified

3) the buyer breached b/f the risk of loss had passed.

·       The seller was a total deficiency in insurance coverage.

·       The main issue in this case is whether the “commercially reasonable time” standard has been satisfied b/c the time between the breach and the destruction of the goods is 1 month.

·       The Court holds that even though there is a 1 month time, this was still a “commercially reasonable time” b/c the buyer continually made statements that they would issue a release. Since the seller relied on the buyer’s statements, the seller had no reason to get insurance.


 

 

VIII. Express Warranties

 

UCC § 2-313, Express Warranties by Affirmation, Promise, Description, Sample

·                     There are three ways that express warranties are created by the seller:

(1)   §2-313(2)(a) Any affirmation of fact or promise made by the seller to the buyer which relates to the goods and becomes part of the basis of the bargain creates an express warranty that the goods shall conform to the affirmation or promise.

·                     Whether an express warranty is created is ascertained by the objective manifestation as received by the buyer. Will NOT depend on the subjective intent of the seller.

·                     Distinguish between statements of fact and mere sales talk (or puffing).

o    In one sense, every statement that a seller makes to a buyer is an opinion. So at what point does the salesperson cross the line into a promise and is no longer an opinion? See subsection (2) below.

·                     The promise or affirmation of fact must be made by the seller to the buyer.

 

(2)   §2-313(2)(b) Any description of the goods which is made part of the basis of the bargain creates an express warranty that the goods shall conform to the description.

·                     The description does NOT have to be made by the seller. If the seller makes the description, then there is an overlap between (1)(a) and (1)(b).

 

(3)   §2-313(2)(c) Any sample or model which is made part of the basis of the bargain creates an express warranty that the whole of the goods shall conform to the sample or model.

·                     Distinguish between a sample and a model. A model creates less of a presumption that it is a literal description of the goods. A sample, taken from the bulk of the goods, is a literal description of the goods. Comment 6.

·                     Prof. Lawrence thinks that it is a waste of time to distinguish between a sample and a model. Ultimately, the main question is “if you are shown something by the seller, it will create express warranties, but as to what characteristics are the warranties created?” “What are the reasonable expectations of an average buyer in the same position as this particular buyer?”

 

·         §2-313(3) -  It is not necessary to the creation of an express warranty that the seller use formal words such as “warrant” or “guarantee” or that he have a specific intention to make a warranty, but an affirmation merely of the value of the goods or a statement purporting to be merely the seller’s opinion or commendation of the goods does not create a warranty.

o    The use of the word “merely” is important b/c when the seller does more than “merely” what is described, the seller has crossed the line into promise.

o    The issue is one of fact as to whether the line has been crossed.

o    The main question is “What are the justifiable expectations of the buyer w/ respect to that statement?

 

 

Autzen v. John C. Taylor Lumber Sales Inc.

·         The seller argued that the Huhta survey did not create an express warranty by description under UCC § 2-313(1)(b) b/c it was not made by the seller.

o    The court rejects this argument b/c the description does NOT have to be made by the seller. The fact that the seller did not make the promise does however, negate UCC § 2-313(1)(a).

·         The seller also argues that the description (survey) was not part of the basis of the bargain b/c the survey came after the contract for the sale of the boat was already made.

o    The court rejects this argument b/c the although the contract was formed as to the price of the boat, the bargain was still ongoing in terms of time of payment and transfer of possession. The survey also created a false sense of security in the buyer as to inspection of the boat and lessened the buyer’s degree of vigilance in inspecting the boat prior to acceptance.

·         The seller also argues that the survey was not part of the basis of the bargain b/c when the seller offered to have the survey done, the buyer indicated that it would not be necessary. Since buyer didn’t care about the survey, it can’t be a part of the basis of the bargain.

o    The court rejects this argument b/c the description does not have to bargained for. Instead, the description must go to the essence of the contract.


IX. Implied Warranties

·         The difference between implied and express warranties is one of degree, not kind. Breach of express warranty doesn’t necessarily mean you have a better case w/ a better recovery.

·         Breach of express warranties will be measured against the words or statements. The breach of implied warranties are measured against the general standards.

·         Why distinguish between express and implied warranties? Why not just have a general warranties category?

o   The Code recognizes the concept of freedom of contract, and allows the parties to disclaim implied warranties.

o   It is extremely difficult to disclaim an express warranty.

 

A. Implied Warranty of Merchantability

 

            UCC § 2-314, Implied Warranty: Merchantability

(1)   Unless excluded or modified (Section 2-316), a warranty that the goods shall be merchantable is implied in a contract for their sale if the seller is a merchant w/ respect to goods of that kind. Under this section the serving for value of food or drink to be consumed either on the premises or elsewhere is a sale.

           

(2)   Goods to be merchantable must be at least such as (must satisfy all):

(a)   pass w/o objection in the trade under the contract description; and

(b)   in the case of fungible (can be intermixed) goods, are of fair average quality w/in the description; and

(c)    are fit for the ordinary purposes for which such goods are used; and

(d)   run, w/in the variations permitted by the agreement, of even kind, quality and quantity w/in each unit and among all units involved; and

(e)    are adequately contained, packaged, and labeled as the agreement may require, and

                                                              i.      Sellers argue that there must be a specific agreement w/ respect to containers, packaging, and labeling b/f the implied warranty of merchantability can be breached.

                                                            ii.      However, most courts will not use the above restrictive interpretation. Comment 10.

(f)    conform to the promise or affirmations of fact made on the container or label if any.  

 

Robert H. Carr & Sons, Inc. v. Yearsley

·       Defendant argues that the “ordinary purpose” of a log chain is not to use it as towing cable. Only to drag logs out of the woods.

·       The court held that it is possible that “log chains” are used for towing trucks, and therefore, denied the defendants demurrer motion. The trial will go on.

·       If the buyer is going to succeed under UCC § 2-314(c), what does the buyer have to prove?

o    The buyer has to prove that other buyers have used the log chains for towing purposes. More industries use the log chain for towing purposes. Testimony by other buyers would be very helpful.

§  The buyer could also use an expert witness who can establish the pattern that the common usage of the log chain is for towing.

o    Suing the seller b/c the log chain failed to “tow a mack truck up a hill” would be a particular purpose which falls under 2-315, not 2-314.

o    The major defense of the sellers is that the product failure was due to owner abuse, and not the defect in the product. That the buyer put more stress on the chain than it was designed to withstand.

 

Webster v. Blue Ship Tea Room, Inc.

·         UCC 2-314(1) applies to this case, as stated by the last sentence of this subsection, “the serving for value of food or drink…”

o    It used to be that service of food and drink was a service and not a sale. However, the Code changes this.

o    However, in order for this subsection to apply, it must be a merchant and the food must be served for value.

·         Plaintiff ordered a bowl of fish chowder, where she then choked on a bone. She sued the restaurant for damages.

 

·         The court held that there are two basic approaches that some courts take:

1) the issue is whether or not the item that caused the injury was foreign or natural to the particular food served.

·         If the P found a chicken bone in the fish chowder, then it would be foreign, and would thus have a cause of action.

·         Prof. Lawrence doesn’t like this test.

2) whether the presence of the object should have been anticipated to have been in the food.

·         The court follows the second approach and holds that since fish chowder involves chunks of fresh fish, then it should be anticipated that a bone might appear. Therefore, the P does not have a cause of action.

Misc. Notes

·         Along w/ the two approaches above, other courts take a third approach:

o    The test should be what might be reasonably expected by the consumer rather than what might be naturally found in the dish.

o    Prof. Lawrence thinks this is the best approach.

o    Ex: the P bites into an olive and cracks his tooth on the pit. Under the natural/foreign test, the P would NOT have a cause of action. But under the reasonable expectations test, if the P looked at the olive and noticed a hole (indicating that it is probably pitted), then the P does have a COA b/c he reasonably expected that there be no pit.

 

Testo v. Russ Dunmire Oldsmobile, Inc.

·         Sale of a used car that was modified for racing. The Plaintiff (buyer) was never told that the car had been used for racing.

·         The D (salesman) argued that the implied warranty of merchantability does not apply in the sale of secondhand articles.

o    The court rejects this argument and holds that the implied warranty applies to used goods b/c UCC 2-314 does not distinguish between new and used.

§  In the comments, the Code only mentions that used goods should be held to a standard that considers that the goods are used. Less can be expected of used goods than if they were new. But there is still an implied warranty of merchantability.

·         The D argues that the used car satisfied the term “merchantable”. The court focuses on UCC 2-314(2)(a) & (c).

·         The court holds that under (c), the car was not fit for ordinary purposes b/c the car would overheat and wouldn’t start after only driving it for 138 miles. The racing modifications made the car not fit for “ordinary purposes”. It made the car good for only “racing purposes.”

·         The court holds that under (a), the car could not pass w/o objection in the trade b/c the car was a “race car”, not a “used car”, as described in the contract.

 

B. Implied Warranty of Fitness for a Particular Purpose

 

            UCC §2-315, Implied Warranty: Fitness for a Particular Purpose

·         §2-315 - Where the seller at the time of contracting has reason to know any particular purpose for which the goods are required and that the buyer is relying on the seller’s skill or judgment to select or furnish suitable goods, there is unless excluded or modified under the next section an implied warranty that the goods shall be fit for such purpose.

o    The hard part to satisfy is the “particular purpose” requirement b/c most of the time, we purchase products for “ordinary purposes.”

o    Also, most of the time, we don’t rely on the seller’s skill to purchase suitable goods.

o    “Particular purpose” has to be one that is distinct from the ordinary purpose OR at least, more specific.

o    It is possible to have an overlap w/ other implied warranties and w/ express warranties.

o    Under UCC 2-315, you can have goods that are not defective (so there is no 2-314 claim), but there is still a 2-315 claim b/c it didn’t satisfy his particular purpose.

 

C. The Warranties of Title and Against Infringement

 

UCC §2-312, Warranty of Title and Against Infringement; Buyer’s Obligation Against Infringement

(1)   Subject to subsection (2) there is in a contract for sale a warranty by the seller that:

(a)   the title conveyed shall be good, and its transfer rightful; and

(b)   the goods shall be delivered free from any security interest or other lien or encumbrance of which the buyer at the time of contracting has no knowledge. “Clean Title”

(2)   A warranty under subsection (1) will be excluded or modified only by specific language or by circumstances which give the buyer reason to know that the person selling does not claim title in himself or that he is purporting to sell only such right or title as he or a third person may have

(3)   Unless otherwise agreed a seller who is a merchant regularly dealing in goods of the kind warrants that the goods shall be delivered free of the rightful claim of any third person by way of infringement or the like but a buyer who furnishes specifications to the seller must hold the seller harmless against any such claim which arises out of compliance w/ the specifications.


 

X. PRIVITY

 

A. Horizontal Privity

·                     Who is the injured party? Perhaps it wasn’t the buyer that was injured. A family of the buyer is the one who wants to bring suit. Historically, none of the family members has a cause of action against the seller b/c there is no vertical privity.

·                     A way to allow the family members to bring suit w/o having vertical privity is through horizontal privity between the buyer and the family member.

 

The privity requirement was first eliminated w/ respect to negligence suits. Courts allowed negligence causes of action even w/o privity w/ the seller or manufacturer.

 

·         There is an evolutionary chain of the removal of the privity requirement to allow more causes of action.

1) Food. Mazzetti.

                  2) Drugs

3) Products that are not ingested but do have close physical contact w/ the body. Ex: cosmetics, clothing, hair products.

 

·         In 1960, the bombshell hit, Bloomfield Motors, which removed the privity requirement for a product (automobile) that has not in close contact w/ the human body nor was it ingested. This case involved HP, b/c the husband purchased the car, but the wife was driving it when she was injured.

·         The UCC doesn’t discuss VP.

·         The UCC deals w/ HP by § 2-318

 

UCC § 2-318, Third Party Beneficiaries of Warranties and Obligations

Alternative A – A seller’s warranty whether express or implied extends to any natural person who is in the family or household of his buyer or who is a guest in his home if it is reasonable to expect that such person may use, consume or be affected by the goods and who is injured in person by breach of the warranty. A seller may not exclude or limit the operation of this section.

·         This is the most restrictive of the three. Has the most requirements.

o    Natural person. Covers only living persons. A corporation is an unnatural person.

o    In the family, household, or guest in the buyer’s home.

§  Some courts held that getting hurt in a car is not a “home”, so no recovery. Some courts allow recovery from injuries in a car.

o    The person involved must have been “reasonably expected to use, consume, or be affected by the goods.”

§  Courts are lenient on this requirement in favor of the plaintiff.

o    Injury to their person.

 

Alternative B – A seller’s warranty whether express or implied extends to any natural person who may reasonably be expected to use, consume or be affected by the goods and who is injured in person by breach of the warranty. A seller may not exclude or limit the operation of this section.

·         This is the middle alternative.

o    Natural person. See above

o    No family, household, or guest requirement.

o    The person involved must have been “reasonably expected to use, consume, or be affected by the goods.” See above

o    Injury to their person. See above

 

Alternative C – A seller’s warranty whether express or implied extends to any person (not only natural, extends to companies) who may be reasonably expected to use, consume or be affected by the goods and who is injured (not only personal injury) by breach of the warranty. A seller may not exclude or limit the operation of this section w/ respect to injury to the person of an individual to whom the warranty extends.

·         This is the least restrictive alternative.

o    Any person, not only natural. Covers corporate entities.

o    The person involved must have been “reasonably expected to use, consume, or be affected by the goods.” See above

o    Injury, not only personal injury.

 

B. Vertical Privity

·                     If the buyer brings suit, who is a legitimate defendant? Historically, it was only the retailer, b/c the buyer didn’t have privity w/ the manufacturer, wholesaler, and distributor.

·                     The main motivation is when the financial worthiness of the immediate seller is in doubt (bankruptcy), then the remote buyer won’t really have a remedy. Thus, he must look to recovery against a distant seller, which historically, VP has denied. But the courts are doing away w/ the VP requirement.

·                     Is the theory of recovery:

1) Strict tort liability;

2) Express warranty; OR

3) Implied warranty?

·                     Is the injury:

1) Personal Injury;

2) Direct Economic Loss; OR

3) Consequential Economic Loss?

 

Revised Article 2

·                     Basically the same as the regular Article 2, in regards to 2-313, 2-314, 2-315, & 2-318, but adds a few things:

1) Remedial Promise – this is when a seller makes a promise that he will do something if a certain condition occurs.

2) Adds two new sections.

3) UCC §2-313A – covers new goods leased/sold to immediate buyer, and then re-sold or re-leased.  Covers any express warranties or remedial promises reasonably expected to be furnished to the buyer upon purchase/lease.  It eliminates the basis of the bargain requirements.  Remedies of 2-313A claims leaves the seller vulnerable to direct and consequential claims but NOT for lost profits claims.

4) UCC 2-313B – it’s comparable to A in that it also only applies to new goods and covers obligations of the seller to the remote buyer.  It’s intended to cover communications (ads/representations) made by the manufacturer, but the remote buyer must have knowledge of that communication and a reasonable expectation that the seller will comply w/ the express warranty.

 

Economic Loss Doctrine Analysis

·                     How much overlap is there between tort and contract? Why might a plaintiff want to make a tort claim, rather than a warranty claim? First of all, torts allows negligence and strict-liability.  But there are several other reasons:

1) There might be no express warranty protection

2) There was an express warranty, but the terms of the warranty have expired

3) Implied warranties were disclaimed

4) The type or extent of relief under breach of warranty might have been expressly limited by the seller in the agreement

5) Privity problems

6) Breach of warranty claim was lost b/c failed to provide the proper notice.

7) Statute of Limitation has run out. Generally, breach of warranty is 4 years and tort is 2 years. However, SOL for warranty runs when the sale is made, SOL for tort doesn’t run until the injury is discovered. So for cases when the injury is discovered 5 years after the sale, then SOL has run out for breach of warranty, so must use tort claim.

 

 

Economic Loss Doctrine – in the 1960’s, defenses arose to warranty claims, forcing buyers to seek relief in torts.

·         There are three approaches used by the courts:

1) Sealy v White Motor Co - Tort liability cannot be applied when the loss is purely monetary. Economic loss recovery only when there is injury to person or other property

2) Allow economic loss recovery when the defect in the product rendered it unreasonably dangerous, even though it didn’t result in physical injury or loss of other property.

3) Allow economic loss recovery when the injury is a sudden, calamitous event, caused by the defect in the product. Rejected by most courts.


 

 

XI. Disclaimers of Warranty

 

UCC § 2-316, Exclusion or Modification of Warranties

(1)   Words or conduct relevant to the creation of an express warranty and words or conduct tending to negate or limit warranty shall be construed wherever reasonable as consistent w/ each other; but subject to the provisions of this Article on parol or extrinsic evidence (Section 2-202) negation or limitation is inoperative to the extent that such construction is unreasonable.

§  A disclaimer of an express warranty seems dishonest at the outset.

§  This subsection serves two functions:

1) Instruct the courts to try to construe the warranty and the disclaimer as consistent w/ each other.

2) However, if the court tries hard to reconcile the two terms so that they are not inconsistent, but cannot do so, the disclaimer is ineffectual.

§  The basic idea is that you cannot disclaim express warranties, b/c it would encourage dishonesty.

 

(2)   Subject to subsection (3), to exclude or modify the implied warranty of merchantability or any part of it the language must mention merchantability and in case of a writing must be conspicuous, and to exclude or modify any implied warranty of fitness the exclusion must be by a writing and conspicuous. Language to exclude all implied warranties of fitness is sufficient if it states, for example, that “There are no warranties which extend beyond the description on the face hereof.”

§  Subsection 2 is the predominant method of disclaimer.

§  A disclaimer of the Implied Warranty of Merchantability does NOT have to be in writing. However, if it is in writing, it must be conspicuous.

§  Regardless of whether it is oral or written, the disclaimer must include the word “merchantability.”

·         Prof. Lawrence thinks the inclusion of the word “merchantability” doesn’t really help buyers b/c they won’t really know what it means.

§  A disclaimer of the Implied Warranty of Fitness for a Particular Purpose must be in writing, but does NOT have to mention any specific writing. Must also be conspicuous.

§  The main question for disclaimer of both of these implied warranties is whether it is conspicuous. Definition of “conspicuous” is in UCC § 1-201(10):

·         Conspicuous means “so written, displayed, or presented that a reasonable person against which it is to operate ought to have noticed it.” Whether a term is conspicuous or not is a decision for the court. Conspicuous terms include the following:

(A) a heading in capitals equal to or greater in size than the surrounding text, or in contrasting type, font, or color to the surrounding text of the same or lesser size, and

(B) language in the body of a record or display in larger type than the surrounding text, or in contrasting type, font, or color to the surrounding text of the same size, or set off from surrounding text of the same size by symbols or other marks that call attention to the language.

·         The standard is an objective standard, where it doesn’t matter whether the buyer actually took notice of the disclaimer, but whether a reasonable person ought to have noticed it.

 

(3)   Notwithstanding subsection (2)

(a)   unless the circumstances indicate otherwise, all implied warranties are excluded by expressions like “as is”, “with all faults” or other language which in common understanding calls the buyer’s attention to the exclusion of warranties and makes plain that there is no implied warranty;

·                     There is also a presumption of the conspicuousness requirement for terms like “as is” although it is not expressly stated in the Code, b/c it would be too unfair for the buyer.

AND

(b)   when the buyer before entering into the contract has examined the goods or the sample or model as fully as he desired or has refused to examine the goods there is no implied warranty w/ regard to defects which an examination ought in the circumstances to have revealed to him; AND

·                     If the buyer is aware of the defect, but has not made an examination of the product, and the buyer goes through w/ the transaction, then the buyer doesn’t have a cause of action, b/c he should have said something about the defect.

 

(c)    an implied warranty can also be excluded or modified by course of dealing or course of performance or usage of trade.

(4)   Remedies for breach of warranty can be limited in accordance w/ the provisions of this Article on liquidation or limitation of damages and on contractual modification of remedy (Sections 2-718 and 2-719).

           

Dorman v. International Harvester Co.

·         Dorman purchased a tractor and backhoe from Harvester. The tractor had problems from the day of delivery and broke down.

·         Dorman brought suit for breach of express and implied warranties of merchantability and fitness for particular purpose, seeking damages for $7,729.98, which includes the contract price and loss of earnings.

·         The court considered the issue of whether the contract for sale contained a valid disclaimer of implied warranties.

o    Dorman testified that he read but did not sign the contract on Oct 31, but later signed but didn’t read the contract on Nov 3, b/c he was told that it was an identical contract. Dorman also didn’t receive a copy of the warranty even though in the contract it stated that he acknowledged receipt of the warranty.

o    In the contract, there is a disclaimer of the warranties of merchantability and fitness for a particular purpose, stating that the manufacturer’s standard printed warranty (which appeared only in the purchase order, not the contract) only applied.

o    The trial court held that the provision was conspicuous and that Dorman should be charged w/ notice of the disclaimer.

§  The disclaimers were in writing, and the one for Merchantability included the word “merchantabilty”.

§  The main issue was whether the disclaimers were conspicuous.

 

·         The court of appeals reversed, holding that the disclaimer provision was not sufficiently conspicuous to have negated the implied warranties. The lettering in the contract was only slightly larger and contrasted to the other paragraph of the contract.

o    “In order to have a valid disclaimer provision, it must be in clear and position as to compel notice.”

o    The contract also did not have a proper heading, such as “DISCLAIMER OF WARRANTIES”.

o    Furthermore, the disclaimer provision is ambiguous and possibly misleading. The words “merchantability” and “fitness for a particular purpose” are italicized, thus possibly causing the reader into thinking that he has such warranties.

o    Also, the manufacturer’s standard warranty was not included in the contract, but was on the reverse side of the purchase order, and a copy was never given to Dorman.

§  “A disclaimer of warranties must be specifically bargained for so that a disclaimer in a warranty given to the buyer after he signs the contract is not binding.”

The best possible argument for seller in Dorman, that the disclaimer is conspicuous is that:

                1) The disclaimer was close to where the buyer signed his name.

2) Italicized lettering

3) The font is slightly larger

4) The font is slightly different

5) Buyer should have read it and in fact, did read it, although he didn’t read the Manufacturer’s standard printed warranty.

6) The disclaimer is set off (lines separating it from the rest of the contract).

The arguments against the seller, made by the court, that the disclaimer is not conspicuous:

                1) The slight difference in size and style of font is not enough to be conspicuous

                2) There is no heading in bold lettering.

3) The italicized words are misleading to the buyer. Only a part of the disclaimer is italicized.

 

XII. Modification of Remedy

·         A disclaimer controls the seller’s liability.

·         The modification of remedy does not affect liability, but affects the remedies that the buyer has after liability has been established. Ex: limit on consequential damages.

 

UCC § 2-719, Contractual Modification or Limitation of Remedy

(1)   Subject to the provisions of subsections (2) and (3) of this section and of the preceding section on liquidation and limitation of damages,

(a)   the agreement may provide for remedies in addition to or in substitution for those provided in this Article and may limit or alter the measure of damages recoverable under this Article, as by limiting the buyer’s remedies to return of the goods and repayment of the price or to repair and replacement of non-conforming goods or parts; AND

·         The most common approach is to limit the remedy to only return of the purchase price or to limit the remedy to “repair or replace”.

·         However, there must still be a minimum adequate remedy and the agreement must be reasonable. Comment 1

 

(b)   Resort to a remedy as provided is optional unless the remedy is expressly agreed to be exclusive, in which case it is the sole remedy.

·         If the seller does not state that it is the exclusive remedy, otherwise, all of the other remedies will still be available along w/ the one that is stated, for example, “repair or replace.” Comment 2

·         This is one way for the buyer to attack the limitation of remedies clause.

 

(2)   Where circumstances cause an exclusive or limited remedy to fail of its essential purpose, remedy may be had as provided in this Act.

·         This is another way for the buyer to attack the limitation of remedies clause.

·         The phrase “its essential purpose” requires a showing that the remedy itself failed, not just that the buyer was not properly compensated.

o    Ex: the consequential damages might be huge, but the “repair and replace” remedy still serves its purpose, so the limitation of remedy is OK.

·         If there is a proper showing that the “remedy fails its essential purpose”, then the limitation provision drops out, and the full range of Article 2 remedies are available. 

 

(3)   Consequential damages may be limited or excluded unless the limitation or exclusion is unconscionable. Limitation of consequential damages for injury to the person in the case of consumer goods is prima facie unconscionable but limitation of damages where the loss is commercial is not.

§  The burden of proof on an unconscionability claim is always on the buyer unless the buyer purchases consumer goods and the consumer goods causes injury to the person.

§  The consequential damages is usually where the largest dollar amounts are involved.


 

XIV. Acceptance

·         The different responses that a buyer can make after seller tenders the goods:

1) Acceptance

2) Rejection – always pre-acceptance behavior.

§  The buyer refuses to take goods b/c they are defective or have arrived too late.

3) Acceptance of the goods followed by Revocation by the buyer. Revocation is always post-acceptance behavior.

§  Acceptance is the pivotal concept b/c it affects whether the buyer can use Rejection or Revocation.

§  The buyer changes his mind about his acceptance and revokes the acceptance b/c the goods are defective.

§  Rejection and Revocation are contrary to what UCC 2-301 states.

§  Rejection and Revocation are referred to as Self-Help remedies.

§  The difference between Rejection and Revocation is going to be in whether the buyer has a “right to do it” or not. Rejection is easy and Revocation is tough.

 

UCC § 2-301, General Obligations of Parties – The obligation of the seller is to transfer and deliver and that of the buyer is to accept and pay in accordance w/ the contract.

·         This section is assuming that there is a valid contract for the sale of goods.

·         To determine whether the buyer has accepted, see UCC § 2-606. If the buyer does not accept, he has a right to reject, and does not have to fulfill the obligation to pay.

 

UCC § 2-606, What Constitutes Acceptance of Goods

(1)   Acceptance of goods occurs when the buyer:

(a)   after a reasonable opportunity to inspect the goods signifies to the seller that the goods are conforming or that he will take or retain them in spite of their non-conformity; OR

·         However, see UCC § 2-513, where if the contract is C.O.D., the buyer must pay even before he has inspected them. The buyer still has the right to reject after inspection, but the problem is that the seller has the buyer’s money.

o    However, payment of the contract price is NOT conclusive as evidence of acceptance.

·         This is active acceptance.

 

(b)   fails to make an effective rejection (subsection (1) of Section 2-602), but such acceptance does not occur until the buyer has had a reasonable opportunity to inspect them; OR

·         This is a form of passive acceptance.

 

(c)    does any act inconsistent w/ the seller’s ownership; but if such act is wrongful as against the seller it is an acceptance only if ratified by the seller.

·         This section is difficult b/c technically, any use of the goods by the buyer would be inconsistent w/ the seller’s ownership. However, certainly not all use by the buyer should be sufficient to indicate an acceptance by the buyer.

o    Some use, for example, can be necessary for the buyer to inspect the goods effectively.

o    Also, some use of the goods after rejection is necessary to mitigate consequential damages, which the buyer has a duty to do.

§  Ex: if the buyer turns around and sells the goods, this is an act that is inconsistent.

 

(2)   Acceptance of a part of any commercial unit is acceptance of that entire unit.

 

UCC § 2-607, Effect of Acceptance; Notice of Breach; Burden of Establishing Breach After Acceptance; Notice of Claim or Litigation to Person Answerable Over

(1)   The buyer must pay at the contract rate for any goods accepted.

·                     However, the buyer still has a remedy under UCC § 2-714, since the seller has still breached the contract. The buyer can sue for the difference in value of what was promised and what was actually delivered.

 

(2)   Acceptance of goods by the buyer precludes rejection of the goods accepted and if made w/ knowledge of a non-conformity cannot be revoked b/c of it unless the acceptance was on the reasonable assumption that the non-conformity would be seasonably cured but acceptance does not of itself impair any other remedy provided by this Article for non-conformity.

 

(3)   Where a tender has been accepted

(a)   the buyer must w/in a reasonable time after he discovers or should have discovered any breach notify the seller of breach or be barred from any remedy; AND

·         The notice does NOT have to be elaborate. The buyer only has to notify the seller that the transaction is still troublesome and must be watched. Comment 4. The seller might have a right to cure the defect, so the buyer must notify him that the transaction is problematic.

·         When the buyer is a merchant:

o    Commercial standards determine what is a reasonable time for notification by the buyer who qualifies as a merchant.

·         When the buyer is a consumer buyer:

o    Many courts hold that a “reasonable time” is more liberal and the time is extended for consumers. Courts are more lenient w/ consumers.

(b)   if the claim is one for infringement or the like (subsection (3) of Section 2-312) and the buyer is sued as a result of such a breach he must so notify the seller w/in a reasonable time after he receives notice of the litigation or be barred from any remedy over for liability established by the litigation.

 

(4)   The burden is on the buyer to establish any breach w/ respect to the goods accepted.

(a)     Preponderance of the evidence standard.

 

(5)   Where the buyer is sued for breach of warranty or other obligation for which his seller is answerable over

(a)    he may give his seller written notice of the litigation. If the notice states that the seller may come in and defend and that if the seller does not do so he will be bound in any action against him by the his buyer by any determination of fact common to the two litigations, then unless the seller after seasonable receipt of the notice does come in and defend he is so bound.

 

(b)   if the claim is one for infringement or the like (subsection (3) of Section 2-312) the original seller may demand in writing that his buyer turn over to him control of the litigation including settlement or else be barred from any remedy over and if he also agrees to bear all expense and to satisfy any adverse judgment, then unless the buyer after seasonable receipt of the demand does turn over control the buyer is so barred.

 

(6)   The provisions of subsections (3), (4) and (5) apply to any obligation of a buyer to hold the seller harmless against infringement or the like (subsection (3) of Section 3-312).

 

Zabriskie Chevrolet, Inc. v. Smith – Part I

·         Sale of a new automobile w/ a defective transmission. The car failed after the buyer had driven 7/10 of a mile from the dealership.

·         The buyer refused to accept the car even after the transmission was replaced. The seller refused to give the buyer a replacement car.

·         Seller argues that the buyer had accepted the goods, so is entitled to the contract price under UCC § 2-607(1).

·         If the buyer is found to have had accepted the car, then he can no longer reject the car, and must only turn to revocation if possible.

·         If the buyer is found to have NOT accepted the car, then the buyer can simply walk away from the agreement by rejection, which is what he wants to do.

·         The court held that when the buyer discovered the defect and rejected the car, the buyer was still exercising the “reasonable opportunity to inspect” under UCC § 2-606(1)(a) & (1)(b) and driving only 7/10 of a mile definitely falls w/in the ambit of a “reasonable opportunity to inspect.” Thus, the buyer had never accepted the goods b/c his reasonable opportunity to inspect had not expired yet.

o    What if the buyer had signed a receipt at the time of delivery that the buyer had found the car to be perfect, but never actually inspected it?

§  It probably wouldn’t change the courts ruling b/c it still doesn’t get around the problem of the buyer having a “reasonable opportunity to inspect.”

§  The court is realistic in that most consumers can’t really inspect cars well until they have driven it for awhile. Thus, even though a buyer signed the receipt, this still doesn’t mean that the buyer had a reasonable opportunity to inspect.

§  The question is not whether the buyer actually inspected the goods, it is whether the buyer had a reasonable opportunity to inspect.

·         Acceptance does not require an actual inspection. Only the opportunity to do so.

o    What if the buyer had actually inspected the car and signed such a receipt?

§  The issue is whether the seller has a legitimate case of the buyer waiving his right to further inspection.

§  It would be very tough for the seller to make this case though, b/c the receipt is probably based on boiler-plate language.

o    What if the buyer had gotten the car home fine and drove the car for a few months, w/ 10,000 mile on the odometer, b/f the defect showed itself (the defect was latent and not caused by the buyer’s use)?

§  This case is up in the air, b/c must determine where to draw the line as to what constitutes a “reasonable opportunity to inspect.”

§  UCC § 2-606 does not require that the buyer actually discover the defect. Only that they had the opportunity to do so. Should we expect each consumer to take apart their car and inspect each part, which is really the only way this defect could have been detected b/f the breakdown?

·         On the flipside, is it too long of a time for the buyer to inspect and expect to get their money back?

·         Most courts would hold that 10,000 miles is too long to fall under a “reasonable opportunity.”

 

Can-Key Industries, Inc. v. Industrial Leasing Corp.

·         The buyer purchased goods from the seller, and then turned around and leased them to a lessee. The buyer purchased the goods on the condition that they were accepted by the lessee.

·         The goods were shipped directly from the seller to the lessee, and the lessee’s receipt of the goods does not constitute acceptance b/c the lessee had to have a “reasonable opportunity to inspect.”

·         The fact that the buyer leased the goods to the lessee doesn’t constitute acceptance by the buyer (even though it is an act inconsistent w/ the sellers) b/c the seller knew about the buyer’s arrangement w/ the lessee beforehand.

o    Ordinarily, this would constitute acceptance, b/c the buyer is performing an act that is inconsistent w/ the seller. However, the buyer in this case protected himself very well and conditioned his acceptance upon the lessee’s acceptance of the leased goods.

·         UCC § 2-606(1)(a) clearly does not apply in this case, b/c the lessee never signified that the goods were conforming or that they would retain them in spite of their non-conformity.

·         UCC § 2-606(1)(b) does not apply b/c the lessee made an effective rejection.

·         The issue is whether the lessee had accepted under UCC § 2-606(1)(c).

o    There are two competing policies: 

1) If the buyer rejected the goods, they should not be able to continue to use them after his rejection.

2) However, the buyer also has a duty of mitigation of consequential damages and should use them for the sake of avoiding economic waste.

·         The court held that the buyer’s use of the goods was NOT an act inconsistent w/ the seller’s ownership b/c the buyer was cooperating w/ the seller to fix the problems, and the use of the goods were tests of whether the problems were fixed.

o    Furthermore, the buyer’s modification and alteration of the equipment is NOT inconsistent w/ the seller’s ownership b/c to hold otherwise would be to penalize the lessee for their good faith effort to fix the defect and to mitigate damages.

o    Prof. Lawrence says that most courts would not be as forgiving when the buyer significantly alters the goods.


 

XV. Rejection

·         UCC 2-601 deals w/ whether the buyer has a right to reject or not.

·         UCC 2-602 outlines the procedure that the buyer must follow when rejecting.

 

UCC § 2-601, Buyer’s Rights on Improper Delivery – Subject to the provisions of this Article on shipment by seller (§2-504) or breach in installment contracts (Section 2-612) and unless otherwise agreed under the sections on contractual limitations of remedy (Sections 2-718 and 2-719), if the goods or the tender of delivery fail in any respect to conform to the contract, the buyer may

            (a) reject the whole; or

            (b) accept the whole; or

(c)    accept any commercial units and reject the remainder.

 

o    This section of the code essentially creates a Perfect Tender Rule.

§  The perfect tender standard applies not only to the goods, but also to the manner of delivery.

§  To safeguard against buyer’s abusing this standard, their remedy will be very small if the goods were not perfect only b/c of a technicality.

 

o   Limitations on UCC 2-601, Perfect Tender Rule

1)      Course of Dealing & Trade Usage – These determine what constitutes perfect tender. Thus, they aren’t really limitations but standards.

2)      Cure under UCC 2-508 (see below) – If the seller has a right to cure the defect, then must still comply w/ perfect tender standard under 2-601.

3)      UCC 2-612 – permits rejection in an installment contract only when the non-conformity substantially impairs the value of the goods to the buyer.

4)      UCC 2-504 – in shipment contracts, only allows rejection of delivery only if material delay or loss ensues. However, this only deals w/ delay situations, but does not deal w/ situations where there are other problems w/ delivery, such as failure to notify the buyer or make a proper contract for transportation of the goods.

5)      UCC 1-203 – the cure right must be exercised in good faith.

                                                              i.      Prof. Lawrence doesn’t think these limitations and exceptions eat up the Perfect Tender Rule.

 

o   Material Breach standard – This is the common law rule as opposed to the Code’s Perfect Tender Rule. Material Breach encompasses the substantial performance standard. They are the antithesis of each other. If there is a material breach, then there can’t be substantial performance, and vice versa.

§  This is a more lenient standard than the Perfect Tender Rule. Can’t cancel the contract unless there is a material breach.

 

UCC § 2-602, Manner and Effect of Rightful Rejection

(1)   Rejection of goods must be w/in a reasonable time after their delivery or tender. It is ineffective unless the buyer seasonably notifies the seller.

§  Failure to do this legally waives the buyer’s right to reject. So they have accepted the goods.

 

(2)   Subject to the provisions of the two following sections on rejected goods (Sections 2-603 and 2-604),

(a)   after rejection any exercise of ownership by the buyer w/ respect to any commercial unit is wrongful as against the seller; AND

(b)   if the buyer has before rejection taken physical possession of goods in which he does not have a security interest under the provisions of this article (subsection (3) of Section 2-711), he is under a duty after rejection to hold them w/ reasonable care at the seller’s disposition for a time sufficient to permit the seller to remove them; BUT

(c)    the buyer has no further obligations w/ regard to goods rightfully rejected.

 

(3)   The seller’s rights w/ respect to goods wrongfully rejected are governed by the provisions of this Article on Seller’s remedies in general (Section 2-703)

 

 

Zabriskie Chevrolet, Inc. v. Smith - Part II

 

Miron v. Yonkers Raceway, Inc.

·                     SEE THE SUPPLEMENT FOR THE FACTS.

·                     The auctioneer is an agent of the seller, and expressly warranted that the horse was sound.

·                     Acceptance is the pivotal concept here.

·                     UCC 2-606(1)(b) is the applicable acceptance provision here. The question is whether the buyer had failed to make a rejection and whether the buyer had a reasonable opportunity to inspect..

(1)     The buyer had a right to reject under UCC 2-601 b/c the horse was non-conforming in that it couldn’t race, which is what the buyer got it for and the seller said that it was good for.

(2)     The rejection by the buyer happened almost immediately (w/in 12 hours) after he discovered that the horse had a broken leg. However, the court held that this was not necessarily timely b/c the horse can get injured while in transit.

(3)     The court held that the buyer did have a reasonable opportunity to inspect b/c other buyers, b/f the auction, brought their vets to inspect the horse (which is customary), while the buyer did not in this case. Thus, under Trade Usage, the buyer did in fact have a reasonable opportunity to inspect. It doesn’t matter that the buyer hadn’t inspected the horse prior to taking possession of the horse. In fact, the buyer should have inspected under Trade Usage, b/c other buyers did w/ their vets.

(4)     Therefore, the court held that the buyer accepted the goods and does not have a right of rejection. The court also held that the buyer also does not have a right to revoke either. The buyer’s only remedy is that he can attempt to recover the damages under UCC 2-714, where he will get the difference in value of the horse as warranted and as delivered.

§  However, the court rejects this remedy, b/c the buyer has the burden of proof (under UCC 2-607) to show that the horse was injured at the time he bought it and was not injured after he took possession.

§  Since the buyer was unable to satisfy this burden, he cannot get damages either.

 

                UCC § 2-603, Merchant Buyer’s Duties as to Rightfully Rejected Goods.

(1)   Subject to any security interest in the buyer (subsection (3) of Section 2-711), when the seller has no agent or place of business at the market of rejection a merchant buyer is under a duty after rejection of goods in his possession or control to follow any reasonable instructions received from the seller w/ respect to the goods and in the absence of such instructions to make reasonable efforts to sell them for the seller’s account if they are perishable or threaten to decline in value speedily. Instructions are not reasonable if on demand indemnity for expenses is not forthcoming.

 

(2)   When the buyer sells goods under subsection (1), he is entitled to reimbursement from the seller or out of the proceeds for reasonable expenses of caring for and selling them, and if the expenses include no selling commission then to such commission as is usual in the trade or if there is none to a reasonable sum not exceeding ten per cent on the gross proceeds.

 

(3)   In complying w/ this section the buyer is held only to good faith and good faith conduct hereunder is neither acceptance nor conversion nor the basis of an action for damages.

 

            UCC § 2-604, Buyer’s Options as to Salvage of Rightfully Rejected Goods

·         §2-604 - Subject to the provisions of the immediately preceding section on perishables if the seller gives no instructions w/in a reasonable time after notification of rejection the buyer may store the rejected goods for the seller’s account or reship them to him or resell them for the seller’s account w/ reimbursement as provided in the preceding section. Such action is not acceptance or conversion.

 

UCC § 2-605, Waiver of Buyer’s Objections by Failure to Particularize

(1)   The buyer’s failure to state in connection w/ rejection a particular defect which is ascertainable by reasonable inspection precludes him from relying on the unstated defect to justify rejection or to establish breach

(a)   where the seller could have cured it if stated seasonably; Or

(b)   between merchants when the seller has after rejection made a request in writing for a full and final written statement of all defects on which the buyer proposes to rely.

o    Policy: of permitting the B to give a quick and informal notice of defects in a tender without penalizing him for omissions in his statement, while at the same time protecting the S who is reasonably misled by the B’s failure to state curable defects.

o    Defect that could have been cured by S:  a B who merely rejcts the delivery without stating his objections to it is probably acting in bad faith and seeking to get out of the deal which has become unprofitable.

o    When the time to cure has past: (1)(b) makes it plain that a S is entitled upon request to a final statement of objections upon which he can rely.

§  What is needed is that he make clear to the B exactly what is being sought.

§  A formal demand under paragraph (b) will be sufficient in the case of a merchant- buyer.

 

(2)   Payment against documents made w/o reservation of rights precludes recovery of the payment for defects apparent on the face of the documents.

·         Subsection (2) applies to the particular case of documents the same principle which the section on effects of acceptance applies to the case of goods.

o    The only defects concerned in the present subsection are defects in the documents which are apparent on their face.

o    Where payment is required against the documents they must be inspected before payment and the payment then constitutes acceptance of the documents.

·         Such acceptance of the documents does not constitute an acceptance of the goods or impair any options or remedies of the buyer for their improper delivery.

 

            UCC § 2-508, Cure by Seller of Improper Tender or Delivery; Replacement

(1)   Where any tender or delivery by the seller is rejected b/c non-conforming and the time for performance has not yet expired, the seller may seasonably notify the buyer of his intention to cure and may then w/in the contract time make a conforming delivery.

·                     This subsection only applies when the seller performs early so that additional time for performance remains.

·                     “Seasonably Notify” in this subsection and in (2) means prompt notification, w/ however, giving the seller enough time to vigilantly claim the right to cure and ascertain the extent of the defect.

·                     The standard for “cure” is still perfect tender. The biggest issue regarding the adequacy of “cure” is whether the seller must replace the nonconforming goods or is entitled to repair them.

o    Repair should be allowed, provided that it will result in goods that conform to the contract.

o    On the other hand, the buyer should not be required to “accept patchwork goods or substantially repaired articles in lieu of flawless merchandise.”

§  If the parties do not specify a time for the seller’s performance, the gap filler provision of section 2-309 must be used.

·         This provides that in absence of agreement, the time for delivery is to be a reasonable time.

·         When the time for delivery is reasonable, either party must notify the other party of an actual proposed performance date being able to establish a breach based on delay.

o    In absence of any prior notification of a proposed delivery date, the buyer cannot reject on the ground that more than a reasonable time for delivery has passed b/c the delivery date is construed as being too indefinite.

o    Even if the parties establish a fixed date for the S’s delivery, they may subsequently modify that date.

§  E.g. after a rightful rejection and the receipt of intention to cure the non conforming tender, the waived the time set in the modification by agreeing to accept additional trees at a later date. Thus, the tender of delivery of the cure was held within the time for the seller’s performance.

 

(2)   Where the buyer rejects a non-conforming tender which the seller had reasonable ground to believe would be acceptable w/ or w/o money allowance the seller may if he seasonably notifies the buyer have a further reasonable time to substitute a conforming tender.

·         This section allows the seller a further reasonable time beyond the K period for performance to cure a nonconforming tender.

o    This time is not indefinie, but rather a “further reasonable time.”

§  Courts should consider “time to be of the essense.”

·         Allowing time to cure is based on balancing the interest of both parties to the K.

o    The B’s right to perfect tender suffers only a marginal inference b/c an effective cure must conform to the K in all respects except the time of the S’s performance.

o    A S; additional time should be viewed as granting a S a second chance, while providing an aggrieved B with performance that is at least close to perfect tender.

·         A breaching S who is granted further reasonable time to substitute a conforming tender must act expeditiously and should apply the necessary attention and effort to correct the deficiency promptly.

o    Delay in investigating and correcting the problem should cause the cure effort to fail .

o    Repeated unsuccessful cure efforts should terminate further cure rights: “opportunity to cure should be contingent on the sellers ability to accomplish it.”

 

§  Reasonable grounds to believe

·         A seller cant qualify for this statutory cure right in the absence of adequate grounds to believe that the tender is perfect or that commercially legitimate reasons justify the belief that the tender neverltheless will not cause the buyer to complain.

o    When sellers known or should have know that a rejection will follow an imperfect tender, they are not entitled to claim surprise at the buyers response or to assert cure rights under §2-508(2). 

o    A seller can satisfy this limitation when a tender made without knowledge of non conformity is made in good faith and is consistent with commercially reasonable standards.

§  Eg. Wilson v. Scampli : merchandise delivered in factory sealed carton = S had reasonable grounds to believe that these sets would be acceptable as delivered.”

·         But if new start up company: then argument probably would not fly.

o    Also, stated assumption not so well founded if seller had received a recall order on the delivered set OR if similar sets had resulted in a significant # of subsequent complaints/ defect.

o    Upgrades: if they give you the better, new improved model:

§  Eg. Model of hearing aid: caused buzzing in ear: had a good basis to reject b/c not what he ordered, so company is going to have to cure and deliver the model that fits what was bargained for. 

·         A seller tendering with knowledge of a deficiency in the tender can still qualify if it a commercially reasonable basis to assume the tender will be acceptable to the B.

o    Joc oil USA:  sellers crude oil with sulfer content of 0.92% under K specifying 0.5% max sulfer content did not preclude sellers right to cure.

§  Had been certified with content of .52% in Italian refinery

§  S believed it would have been acceptable due to prior knowledge of Con Ed purchase and use practices.

o    Zabriskie: S did not qualify under the reasonable grounds requirement  b/c the sellers mechanics, inspection was supposed to be done on the transmission (in 70 point inspection test), so mechanics either knew or should have known about substantial defects in the cars sold. (Need to prove that known or should have known).

§  Lawrence: this is too sweeping: courts holding on the sellers belief of acceptability is sustainable only if the transmission defect was a type that should have been discovered through a reasonable dealer inspection of the component.

·         Could be a lot of internal latent defects that could not show up in the 70 point inspection.

 

·         Money Allowance

o    Buyer is entitled to what he bargained for

§  Even for time for performance has passed, have time to make conforming tender     

·         Money allowance instead is not conforming tender

o    If the S had reasonable grounds to believe that the B would agree to a price allowance for the nonconforming aspect of the sellers tender a rejection made by the buyer would legitimately surprise the seller.

§  The seller thus could qualify for an opportunity to cure the tender by providing seasonable notification of an intent to cure.

§  See Joc Oil (210-211).

·         S also made argument that even if they knew at time that sulfer was .92% it would still tender that shipment to Con Edison b/c reason to believe that would take oil with money allowance b/c con Edison was producing oil with precisely that nature.

o    So had reason to believe and nevertheless would be acceptable to Con Ed with money allowances

§  Since con ed said “no” under this section has reasonable right to cure; extended time. 

·         Conforming Tender

o    Conforming: “if they are in accordance with obligations under the K” 2-601.

§  Must be in perfect tender in all aspects of the K specifications; except that an additional reasonable time is allowed when the cure is made pursuant to section 2-508(2).

·         Further reasonable time: Lawrence would advice clients: time is of the essence: prompt attention; not to wait around.

 

o    Only way to get into 2-508 is rejection

o    3 options here

§  (1) Seller will not go forward with the cure within the further reasonable time allowed

·         Here further action will not be necessary for an effective rejection

§  (2) S will provide a conforming tender within a further reasonable time.

·         a new conforming tender will cure the original nonconformity and terminate the B’s right to reject.

§  (3) S will provide a non conforming tender.

·         Initial rejection covers the 1st tender: the B must respond to the subsequent tender or ineffective cure and failure to do so after a reasonable time to inspect the newly tendered goods will result in their acceptance by the buyer.

 

 

·         Repair

o    Should be allowed provided that it will result in goods that conform to the K. If quality of the product is not degraded and it is acceptable as a product that never had the deficiency, cure in form of repair is adequate.

§  The repaired product must comply with all the applicable warranties for the repair to satisfy the perfect tender rule and to constitute and adequate cure.

§  Zabriskie: dealer replaced the defective transmission with one that was removed from another vehicle on the dealer’s showroom floor.

·         Dealer replaced transmission from unknown lineage: Lawrence said this is bullshit. Cant see how could put in something other than another Chevy transmission designed from that other car.

·         Π wanted a new model.

 

o    Shaken faith doctrine:: considers the effect that knowledge of the original defect has on a disappointed buyer.

§  When knowledge of the circumstances of repair would leave a reasonable buyer sufficiently apprehensive about the reliability or quality of the purchased product, the B does not receive what was bargained for and the repair is not an adequate cure. .

§  Acceptability of the repair as a cure should thus be determined from the perspective of a reasonable buyer.

§  Problem with the shaken faith doctrine in this case.  However can argue that: S completely replaced transmission and that the defect was not anything more than an isolated instance and that the replacement would able the car to performs represented.

 

§  No internal impact to other parts and components as in Bayne v. Nall Motors, inc.

(bottom of page 213), problem of axle going 85 mph could have caused damage to the whole car that replacing the axle could very well possibly fix the rest of the car: shaken faith doctrine applies here.

 

§  Wilson v. Scampoli: Issue: can a seller repair the goods or are they required to replace the goods

·         They are required to substitute it for a conforming tender.

·         If you use repair: can’t pass of patchwork goods.

·         In this case, the S was not given an adequate determination if the cause was a minor or a substantial defect, B just wanted a brand new TV and didn’t even accept a cure.

·         Since S was denied reasonable opportunity to repair; B interfered with right and tried to dictate method of conforming tender. B has a duty of good faith to cooperate with S to make a reasonable inspection of goods to determine if need new set or simply replace a minor defect to make the good conforming.

·         Not allowing to make determination = B losing effectiveness to rejection.

·         Consensual Care

o    Can arise any time following the S’s initial tender.

o    Parties who reach their cure agreement can shape their cure arrangement as they please on terms concerning the nature of the corrective action or the length of time that will be allowed to cure.

o    Niether party can then revert unilaterally to the original terms of the K.

§  Modification agreement redefines the sellers required performance under the K.

 

Procedure of Cure

1) Effective rightful rejection by the buyer

2) Notice of intent to cure by the seller if seller has a right to cure, under §2-508(1) or (2) or some other code provision.

3) One of three things happens:

a) If the seller never gets around to curing w/in a reasonable time, then the buyer’s rejection stands.

b) If the seller does cure, and the buyer still rejects, then the buyer is in breach of contract.

c) If the seller does cure, but the cure does not satisfy 2-601, then the buyer must again reject the second tender, b/c if he does not, then he has accepted b/c he didn’t make an effective rejection.

 

Wilson v. Scampoli

o    However, Prof. Lawrence states that “factory sealed cartons” should not be conclusive in determining whether the seller had reasonable grounds to believe.

o    Ex: If the seller received a TV in a factory sealed carton but was informed by the manufacturer that this model of TV has had problems, then the seller doesn’t have reasonable grounds to believe that the goods would be OK.

·                     “There is no mandate to require the buyer to accept patchwork goods.” However, this does NOT rule out repair of goods as the method of cure. The seller only has to provide a perfect tender. If the seller can provide a perfect tender through repair, then the seller’s cure can be repair.

 

·                     The court holds that b/c the seller was never given a chance to determine the extent of the defect and whether the defect required repair or replacement, the buyer interfered w/ the seller’s right to cure after the seller gave notice of the intent to cure. Therefore, the contract is not rescinded and the buyer does NOT get the return of the purchase price since the buyer lost the effectiveness of her rejection.

 

Zabriskie Chevrolet, Inc. v. Smith – Part III

o    Prof. Lawrence thinks that the court’s reasoning in this case is wrong. He doubts that the seller knew about the defect and doubts that the seller should have known about it b/c certain latent defects in the transmission are hard to discover, even by thorough inspection.

·                     The court also holds that the seller, even if they did have a right to cure, didn’t satisfy the standard under 2-601 b/c the substitute transmission was taken from another car, which might not have been a new transmission.

o    Prof. Lawrence thinks that this is wrong b/c the seller took a brand new transmission from another car, which should satisfy the perfect standard rule.

·                     The Smiths demanded a different type of cure. Rather than repair, the Smiths wanted replacement of the car.

o    The court held that the Smiths were entitled to demand replacement and reject repair of the cars b/c of the “Shaken Faith Doctrine”, where when dealing w/ a new car, a buyer needs to have faith in the car’s dependability and safety. By repairing the car, the buyer would still have shaken faith, and thus, the repair would not satisfy.

§  Prof. Lawrence thinks the doctrine is OK, but courts have incorrectly applied the doctrine.

§  The Smiths are trying to dictate the method of cure, and the Smiths don’t have that option. The seller’s only obligation under cure is to provide perfect tender and the seller was trying to do that.

§  If the seller has a legitimate right to cure (which the court held that they didn’t), then the seller only has to provide perfect tender through their cure. The question is whether repair can be perfect tender. The court held that under the “shaken faith doctrine”, repair cannot be perfect tender when dealing w/ a brand new car.


 

XVI. Revocation

 

            UCC § 2-608, Revocation of Acceptance in Whole or in Part

(1)   The buyer may revoke his acceptance of a lot or commercial unit whose non-conformity substantially impairs its value to him if he has accepted it:

·         Unlike the right to reject (perfect tender), revocation’s “substantial impair” standard is like the common law “material breach” standard.

·         Whether the value is substantially impaired is determined by each buyer’s personal circumstances. Subjective standard. The same defect could affect two different buyers very differently.

 

(a)   on the reasonable assumption that its non-conformity would be cured and it has not been seasonably cured; OR

(b)     Subsection (1)(a) deals w/ situations where the buyer accepts after having already discovered the defect.

 

(b)   w/o discovery of such non-conformity if his acceptance was reasonably induced either by the difficulty of discovery b/f acceptance or by the seller’s assurances.

(c)     Subsection (1)(b) deals w/ situations where the buyer accepts w/o having discovered the nonconformity.

 

(2)   Revocation of acceptance must occur w/in a reasonable time after the buyer discovers or should have discovered the ground for it AND before any substantial change in condition of the goods which is not caused by their own defects. It is not effective until the buyer notifies the seller of it.

·         Ex: the buyer purchases a barrel of apples, but there is one rotten apple at the bottom, which causes the surrounding apples to also rot. B/c there has been a substantial change in the condition of goods which is caused by the defect, the buyer does not have to revoke acceptance w/in a reasonable time.

·         The “reasonable time” standard is usually pretty long for buyers b/c revocation usually only occurs after attempts at adjustment have failed.

 

(3)   A buyer who so revokes has the same rights and duties w/ regard to the goods involved as if he had rejected them.

·         Unlike after a rejection, the seller does NOT have a right to cure under a revocation.

 

 

Miron v. Yonkers Raceway

·         The court held that the buyer does not have a right to revoke b/c he can’t comply w/ UCC 2-608(1)(a) nor (b).

o    (a) There was no reasonable assumption that the seller would cure the defect since a broken leg for a horse is not fixable.

o    (b) The buyer’s acceptance was not reasonably induced by the difficulty of discovery b/c the buyer only needed to have a veterinarian examine the horse b/f accepting.

§  Furthermore, the even though the seller’s agent (auctioneer) did make assurances, it certainly is not strong enough to overcome the trade usage of buyers having their vets examine horses b/f purchase.

 

Zabriskie Chevrolet, Inc. v. Smith

·         If the buyer discovers the defect b/f acceptance, then he must comply w/ 2-608(1)(a). The buyer must have reasonable assumption that the defect would be cured.

·         If the buyer discovers the defect after acceptance, then he must comply w/ 2-608(2). The buyer must have been reasonably induced either by the difficulty of discovery b/f acceptance or by the seller’s assurances.

·         If the buyer wrongfully revokes an acceptance, the cases are split as to what the consequences. Most courts hold that wrongful revocations are not legally effective.


 

XVII. Breach and Impaired Expectations

 

A. Anticipatory Breach

·         The common law is that when making an agreement, the parties have certain obligations and expectations and each party will perform their obligations. Also, each party is entitled to expect that the other party will not impair your expectations.

o   The most serious impairment of expectations comes from anticipatory repudiation. In any contract, there is an implied promise that neither party will repudiate.

·         The UCC takes the same approach as the common law w/ respect to Anticipatory Breach. The main difference between the UCC and the common law is where a party’s actions don’t quite amount to an Anticipatory Breach, but only amounts to Prospective Inability to Perform.

·         Under PIP, there is not yet a breach. But under the common law, there is an implied condition that at all times from the making of the contract to the time for performance, both parties must be ready at all times to perform their obligations.

o   However, each of their obligations are constructively conditioned on each other’s ability to perform. Thus, X does not have to be ready to pay unless Y is ready to work.

o   If there is a statement of doubt by X, that is pretty serious about his inability to perform, then Y’s obligation to pay is excused, b/c the obligations are constructively conditioned on one another.

§  If the aggrieved party invokes the PIP doctrine, the aggrieved party in this instance, cannot bring a lawsuit b/c there was no breach. Y was excused from his obligation to pay.

·         PIP is NOT relevant to UCC Article 2, only the common law. UCC 2-609 deviates from PIP.

 

UCC § 2-610, Anticipatory Breach

When either party repudiates the contract w/ respect to a performance not yet due the loss of which will substantially impair (this is similar to material breach, not perfect tender) the value of the contract to the other, the aggrieved party may…

(a)   for a commercially reasonable time await performance by the repudiating party; OR

§  Hold off suing for damages.

 

(b)   resort to any remedy for breach (Section 2-703 or Section 2-711), even though he has notified the repudiating party that he would await the latter’s performance and has urged retraction; AND

§  Suing for damages.

§  2-703 and 2-711 indicates the right to cancel the contract.

 

(c)    in either case suspend his own performance or proceed in accordance w/ the provisions of this Article on the seller’s right to identify goods to the contract notwithstanding breach or to salvage unfinished goods (Section 2-704).

 

UCC § 2-609, Right to Adequate Assurance of Performance

(1)   A contract for sale imposes an obligation on each party that the other’s expectation of receiving due performance will not be impaired. When reasonable grounds for insecurity arise w/ respect to the performance of either party the other may in writing demand adequate assurance of due performance and until he receives such assurance may, if commercially reasonable, suspend any performance for which he has not already received the agreed return.

 

(2)   Between the merchants the reasonableness of grounds for insecurity and the adequacy of any assurance offered shall be determined according to commercial standards.

 

(3)   Acceptance of any improper delivery or payment does not prejudice the aggrieved party’s right to demand adequate assurance of future performance.

 

(4)   After receipt of a justified demand, failure to provide w/in a reasonable time not exceeding thirty days such assurance of due performance as is adequate under the circumstances of the particular case is a repudiation of the contract.

 

Graulich Caterer, Inc. v. Hans Holterbosch, Inc.

·         There was breach b/c food delivered was bad, and did not come close to matching the samples, and did not satisfy the Warranty of Fitness for a Particular Purpose.

·         The perfect tender rule of 2-601 is not applicable to installment contracts, which are governed by 2-612.

·         The buyer’s rejection of the first installment is governed by UCC 2-612(2), and the rejection of the second installment (the cure) is governed by UCC 2-612(3).

·         The food in this case was non-conforming and could not be cured, so rejection was proper under 2-612(2). The second installment was the seller’s cure attempt, which also failed.

·         Rejection under 2-612(3) of the remaining installments was proper b/c non-conformity of the first two installments “substantially impaired the value of the whole contract.” Time was critical. The buyer had an immediate need for quality food and the seller was unable to cure. Thus, the buyer rightfully rejected b/c there was no reason for the buyer to think that the seller would cure the defect in time for the buyer to serve food soon after.

 

UCC § 2-611, Retraction of Anticipatory Repudiation

(1)   Until the repudiating party’s next performance is due he can retract his repudiation unless the aggrieved party has since the repudiation cancelled or materially changed his position or otherwise indicated that he considers the repudiation final.

 

(2)   Retraction may be by any method which clearly indicates to the aggrieved party that the repudiating party intends to perform, but must include any assurance justifiably demanded under the provisions of this Article (Section 2-609)

 

(3)   Retraction reinstates the repudiating party’s rights under the contract w/ due excuse and allowance to the aggrieved party for any delay occasioned by the repudiation.

 

UCC § 2-612, Installment Contracts Breach

(1)   An “installment contract” is one which requires or authorizes the delivery of goods in separate lots to be separately accepted, even though the contract contains a clause “each delivery is a separate contract” or its equivalent.

 

(2)   The buyer may reject any installment which is non-conforming if the non-conformity substantially impairs the value of that installment and cannot be cured or if the non-conformity is a defect in the required documents; but if the non-conformity does not fall w/in subsection (3) and the seller gives adequate assurance of its cure the buyer must accept that installment.

 

(3)   Whenever non-conformity or default w/ respect to one or more installments substantially impairs the value of the whole contract there is a breach of the whole. But the aggrieved party reinstates the contract if he accepts a non-conforming installment w/o seasonably notifying of cancellation or if he brings an action w/ respect only to past installments or demands performance as to future installments.

 

 

 

UCC § 2-703, Seller’s Remedies in General

(1)   Where the buyer wrongfully rejects, revokes, fails to make a payment due on or before delivery, fails to perform a K obligation or repudiates, it is a breach. 

 

(2)   If the buyer is in breach to the seller, the seller may

(a)   withhold delivery of such goods;

(b)   stop delivery of the goods (§ 2-705);

(c)    proceed under the next section respecting goods still unidentified to the contract;

(d)   reclaim the goods §2-507(2) or 2-702(2)

(e)    Require payment directly from the buyer §2-325(c)

(f)    cancel

(g)   resell and recover damages as hereafter provided (§ 2-706);

(h)   recover damages for non-acceptance or repudiation (§ 2-708)

(i)     recover lost profits under §2-708(2)

(j)     recover the price under §2-709

(k)   obtain specific performance under §2-716

(l)     recover liquidated damages under §2-718

(m) in other cases, recover damages in any manner that is reasonable under the circumstances

 

UCC § 2-711, Buyer’s Remedies in General; Buyer’s Security Interest in Rejected Goods

(1)   A breach of K by the S includes the Seller’s wrongful failure to deliver or to perform a contractual obligation, making of a non-conforming tender of delivery or performance, and repudiation.

 

(2)   If the seller is in breach of contract under (1), the buyer, to the extent provided for by this Act or other law, may:

(a)   in the case of rightful cancellation, rightful rejection, or justifiable revocation of acceptance, recover so much of the price as has been paid

(b)   deduct damages from any part of the price still due under §2-717

(c)    cancel

(d)   “cover” and have damages under §2-712 as to all goods affected whether or not they have been id’d to the K

(e)    recover damages for nondelivery or repudiation under §2-713

(f)    recover damages for breach w/regard to accepted goods or breach w/regard to a remedial promise under §2-714

(g)   recover identified goods under §2-502.

(h)   obtain specific performance or obtain the goods by replevin or similar remedy under §2-716

(i)     recover liquidated damages under §2-718

(j)     in other cases, recover damages in any manner that is reasonable under the circumstances

(3)   On rightful rejection or justifiable revocation of acceptance a buyer has a security interest in goods in his possession or control for any payments made on their price and any expenses reasonably incurred in their inspection, receipt, transportation, care and custody and may hold such goods and resell them in like manner as an aggrieved seller (Seller 2-706).


 

XVIII. Remedies of Buyers

·         Compare the availability of the buyer’s remedies:

a) Either UCC §2-712 (Cover) or UCC §2-713 (Market Price);

§  These remedies are used in the case of non-delivery, anticipatory repudiation, buyer rightfully rejects or rightfully revokes.

§  If the buyer covers, then can’t use 2-713.

§  The Code prefers the Cover remedy over the Market Price remedy b/c the Cover remedy is more of an accurate remedy in putting the buyer in the same position as if the contract had been performed.

§  Must also consider UCC §2-715 (Incidental & Consequential damages)

 

OR

 

b) UCC §2-714 (Remedy when Accepting Goods)

§  Only can be used when the buyer has accepted the goods. If buyer accepts, can’t use 2-712 nor 2-713.

§  Must also consider UCC §2-715 (Incidental & Consequential damages)

 

A. Cover

UCC § 2-712, “Cover”; Buyer’s Procurement of Substitute Goods

(1)   After a breach w/in the preceding section the buyer may “cover” by making in good faith and w/o unreasonable delay any reasonable purchase of or contract to purchase goods in substitution for those due from the seller.

o    The goods purchased during cover do not have to be identical, only that they are “commercially usable as reasonable substitutes under the circumstances of the particular case.” Comment 2

o    The test of proper cover is whether at the time and place the buyer acted in good faith (“honesty in fact and w/in the boundaries of reasonable commercial standards”) and in a reasonable manner, and it is immaterial that hindsight may later prove that the method of cover was not the cheapest or most effective. Comment 2

o    See Problem #2

o    If the buyer does not make a qualifying cover (if any one of these requirements is not met) then must go to 2-713 (market price/contract price differential). It would be the same as if the buyer had not covered at all.

 

(2)   The buyer may recover from the seller as damages the difference between the cost of cover and the contract price together w/ any incidental or consequential damages as hereinafter defined (Section 2-715), but less expenses saved in consequence of the seller’s breach.

 

(3)   Failure of the buyer to effect cover w/in this section does not bar him from any other remedy.

 

B. Market Price/Contract Price Differential

UCC § 2-713, Buyer’s Damages for Non-Delivery or Repudiation

(1)   Subject to the provisions of this Article w/ respect to proof of market price (Section 2-723), if the seller wrongfully fails to deliver or repudiates or the buyer rightfully rejects or justifiably revokes acceptance:

(a)   the measure of damages for wrongful failure to deliver by the seller or rightful rejection or justifiable revocation of acceptance by the buyer is the difference between the market price at the time for tender and the contract price together w/ any incidental and consequential damages provided in this Article (Section 2-715), but less expenses saved in consequence of the seller’s breach; and

 

(b)   the measure of damages for repudiation by the seller is the difference between the market price at the expiration of a commercially reasonable time after the buyer learned of the repudiation, but no later than the time stated in (a), and the K price together w/any incidental or consequential damages provided in §2-715, less expenses saved in consequence of the seller’s breach.

·         This remedy reflects the common law approach.

·         If the buyer has NOT accepted the goods and the buyer has not covered, this is the appropriate remedy. The buyer does not have to cover and can decide to use the market price remedy (2-713).

·         Must establish the “where” and “what time” of the market.

o    The time of the market price is “at the time when the buyer learned of the breach.”

§  This raises problems w/ proving of when the buyer learned of the breach.

§  The main problem is when dealing w/ anticipatory repudiation. However, Prof. Lawrence doesn’t want us to go over this b/c it is taken care of in the amendments.

·         For market price remedy calculations, Prof. Lawrence doesn’t want us to spend too much time of finding “incidental & consequential damages” and “less expenses saved.”

 

(2)   Market price is to be determined as of the place of tender or, in cases of rejection after arrival or revocation of acceptance, as of the place of arrival.

·                     See Problem below.

o    First, determine whether the goods were not delivered at all, or whether they were delivered and then rejected (subsection (2) of 2-713 then applies).

o    Second, determine whether the contract is a shipment or destination contract.

 

C. When Buyer Accepts the Goods

UCC § 2-714, Buyer’s Damages for Breach in Regard to Accepted Goods

(1)   Where the buyer has accepted goods and given notification (subsection (3) of Section 2-607) he may recover as damages for any non-conformity of tender the loss resulting in the ordinary course of events from the seller’s breach as determined in any manner which is reasonable.

o    If the acceptance is revoked b/f the cause of action, then 2-714 does not apply, b/c the acceptance disappeared.

o    There must be an acceptance that is still effective (not revoked) at the time of the cause of action in order to use the 2-714 remedy.

o    Ex: late delivery, tender of non-conforming documents.

 

(2)   The measure of damages for breach of warranty is the difference at the time and place of acceptance between the value of the goods accepted and the value they would have had if they had been as warranted, unless special circumstances show proximate damages of a different amount.

o    The value of the goods as warranted is often determined by the contract price, but it is not conclusive. If the buyer can prove that the value of the goods exceeds the contract price, then the buyer should get the benefit of the bargain. Or if the value of the goods is less than the contract price, then the buyer should be stuck w/ the foolishness of the bargain.

o    Cost to repair is often the measure of the difference.

 

(3)   In a proper case any incidental and consequential damages under the next section may also be recovered.

 

D. Incidental and Consequential Damages

UCC § 2-715, Buyer’s Incidental and Consequential Damages

(1)   Incidental damages resulting from the seller’s breach include expenses reasonably incurred in inspection, receipt, transportation and care and custody of goods rightfully rejected, any commercially reasonable charges, expenses or commissions in connection w/effecting cover and any other reasonable expense incident to the delay or other breach.

§  Incidental damage is designed to cover two types of costs:

1) Expenses incurred while the aggrieved buyer is fulfilling their obligations under the contract, where the buyer is “following the reasonable instructions of the seller while holding the goods for the seller.”

2) Expenses incurred while exercising the buyer’s own rights under the contract. Ex: inspection of defective goods or transactional costs for cover.

 

(2)   Consequential damages resulting from the seller’s breach include

(a)   any loss resulting from general or particular requirements and needs of which the seller at the time of contracting had reason to know and which could not reasonably be prevented by cover or otherwise; AND

§  But-For Test – “But for the seller’s breach, I would not have suffered this damage.”

·         However, there is a foreseeability requirement. “The seller must have either known about it or had reason to know about it at the time of contracting.” Hadley v. Baxendale.

§  Although not stated in the Code, the buyer must be able to prove consequential damages w/ reasonable certainty. It doesn’t have to be exact, but speculation is not good enough.

·         However, the “established business rule”, where only established businesses can receive damages for lost profits, has been generally rejected by the courts.

§  Mitigation Principle – “could not reasonably be prevented by cover or otherwise.” The buyer is under a duty to mitigate by covering or other methods. If the buyer could have mitigated any of the loss, but did not, the buyer cannot recover the losses for which the buyer could have mitigated.

 

(b)   Injury to person or property proximately resulting from any breach of warranty.

§  This is the proximate cause requirement. The torts standard BUT NO FORSEEABILITY REQUIREMENT.

·         Even though there may be actual cause, it doesn’t necessarily mean that there is proximate cause.

§  Note that the foreseeability principle (Hadley v. Baxendale) is not present in this subsection.

§  Lewis v. Mobile Oil Corporation – The buyer purchased a lubricant for their machinery from the local oil dealer. He asked the seller which type the buyer needed, and the seller made a recommendation. Ultimately, the lubricant used was the wrong kind and the machines were badly damaged. Buyer brought a COA for breach of warranty.

·         Breach of Implied Warranty of Fitness for a Particular Purpose.

·         As consequential damages, the buyer received the cost of repair and replacement of the parts that were damaged as well as lost profits during the repair time of the machine.

 

E. Specific Performance

UCC § 2-716, Buyer’s Right to Specific Performance or Replevin

(1)   Specific performance may be decreed where the goods are unique or in other proper circumstances.

§  “Unique” does not mean only “one of a kind.” Goods can be unique even if there is more than one (ex: limited edition corvette).

§  One example of “other proper circumstances” is when the buyer is unable to cover. This is the only example given in the comments.

 

(2)   The decree for specific performance may include such terms and conditions as to payment of the price, damages, or other relief as the court may deem just.

 

(3)   The buyer has a right of replevin for goods identified to the contract if after reasonable effort he is unable to cover for such goods or the circumstances reasonably indicate that such effort will be unavailing or if the goods have been shipped under reservation and satisfaction of the security interest in them has been made or tendered.

 

(4)   The buyer’s right under (3) vests upon acquisition of a special property, even if the seller had not then repudiated or failed to deliver.

·         If there is an inability to cover and the goods are identified, the courts must award a right to replevin. Unlike specific performance, where the court’s decision is discretionary (“may”), the courts decision in right to replevin, if the goods are identified and the buyer is unable to cover, then the court is compelled to grant a right to replevin.


 

XIX. Remedies of Sellers

A. Resale (analogous to Buyer’s Cover)

UCC § 2-706, Seller’s Resale Including Contract for Resale

(1)   Under the conditions stated in Section 2-703 on seller’s remedies, the seller may resell the goods concerned or the undelivered balance thereof. Where the resale is made in good faith and in a commercially reasonable manner the seller may recover the difference between the resale price and the contract price together w/ any incidental damages allowed under the provisions of this Article (Section 2-710), but less expenses saved in consequence of the buyer’s breach.

 

(2)   Except as otherwise provided in subsection (3) or unless otherwise agreed resale may be made at public or private sale including sale by way of one or more contracts to sell or of identification to an existing contract of the seller. Sale may be as a unit or in parcels and at any time and place on any terms but every aspect of the sale including the method, manner, time, place and terms must be commercially reasonable. The resale must be reasonably identified as referring to the broken contract, but it is not necessary that the goods be in existence or that any or all of them have been identified to the contract b/f the breach.

 

(3)   Where the resale is at private sale the seller: must give the buyer reasonable notification of his intention to resell.

 

(4)   Where the resale is at public sale

(a)   Only identified goods can be sold except where there is a recognized market for a public sale of futures in goods of the kind; AND

 

(b)   it must be made at a usual place or market for public sale if one is reasonably available and except in the case of goods which are perishable or threaten to decline in value speedily the seller must give the buyer reasonable notice of the time and place of the resale; AND

 

(c)    if the goods are not to be w/in the view of those attending the sale the notification of sale must state the place where the goods are located and provide for their reasonable inspection by prospective bidders; AND

 

(d)   the seller may buy.

 

(5)   A purchaser who buys in good faith at a resale takes the goods free of any rights of the original buyer even though the seller fails to comply w/ one or more of the requirements of this section.

 

(6)   The seller is not accountable to the buyer for any profit made on any resale. A person in the position of a seller (Section 2-707) or a buyer who has rightfully rejected or justifiably revoked acceptance must account for any excess over the amount of his security interest, as hereinafter defined (subsection (3) of Section 2-711).

 

(7)   Failure of a seller to resell under this section does not bar the seller from any other remedy.

 

B. Contract Price/Market Price Differential (analogous to Buyer’s remedy of same name)

            UCC § 2-708, Seller’s Damages for Non-Acceptance or Repudiation

(1)   Subject to subsection (2) and to the provisions of this Article w/ respect to proof of market price (Section 2-723):

(a)   the measure of damages for non-acceptance by the buyer is the difference between the market price at the time and place for tender and the unpaid contract price together w/ any incidental damages provided in this Article (Section 2-710), but less expenses saved in consequence of the buyer’s breach.

(b)   the measure of damages for repudiation by the buyer is the difference btw the K price and the market price at the place for tender at the expiration of a commercially reasonable time after the seller learned of the repudation, but no later than the time stated in (a), together w/any incidental or consequential damages provided in §2-710, less expenses saved in consequence of the buyer’s breach.

 

C. Lost Profits

UCC § 2-708, Seller’s Damages for Non-Acceptance or Repudiation

(2)   If the measure of damages provided in subsection (1) is inadequate to put the seller in as good a position as performance would have done then the measure of damages is the profit (including reasonable overhead) which the seller would have made from full performance by the buyer, together w/ any incidental damages provided in this Article (Section 2-710), due allowance for costs reasonably incurred and due credit for payments or proceeds for resale.

(1)     Lost Volume Seller – Under the 7th Circuit, there are three requirements for a seller to qualify as a lost volume seller:

1) the seller must possess the capacity to make an additional sale;

2) the additional sale would have been profitable for the seller; AND

3) the seller probably would have made the additional sale even if the breach had not occurred.

 

D. Action for Price (this is sort of like Buyer’s remedy of Specific Performance)

·         This remedy is a court judgment that orders the buyer to pay the contract price.

·         If the buyer refuses to pay, then the court executes their order and gets the sheriff to seize the buyer’s property (lien) to pay off the seller and pay the sheriff’s expenses.

·         This is not the preferred remedy. The preferred remedy is UCC 2-706, Resale.

 

 

UCC § 2-709, Action for the Price

(1)   If the buyer fails to pay the price as it becomes due the seller may recover, together w/ any incidental damages under the §2-710, the price

(a)   of goods accepted or conforming goods lost or damaged w/in a commercially reasonable time after risk of their loss has passed to the buyer; AND

§  Generally, seller can not get action for the price if the buyer has not accepted the goods. Even if the buyer wrongfully rejects, the wrongful rejection still precludes any acceptance, and the seller will have difficulty getting action for the price.

·         The policy behind this is that seller’s who have control of the goods are in a better position to resell them.

§  However, an exception to the buyer’s acceptance requirement is when the goods are lost or damaged after the risk of loss has passed to the buyer.

·         When goods are lost or damaged, then the policy behind the buyer’s acceptance requirement no longer applies.

·         Must consider who has the risk of loss. Shipment or destination contract.

 

(b)   of goods identified to the contract if the seller is unable after reasonable effort to resell them at a reasonable price or the circumstances reasonably indicate that such effort will be unavailing.

·         This is another exception to the buyer’s acceptance requirement. This subsection undercuts the policy behind the buyer’s acceptance requirement.

·         Seller’s should document their efforts to resell.

·         A situation where “such efforts will be unavailing” is when technology makes the goods obsolete or totally unwanted. If the goods are identified to the contract, then can get action for the price.

 

(2)   If the seller sues for the price he must hold for the buyer any goods which have been identified to the contract and are still in his control except that if resale becomes possible he may resell them at any time prior to the collection of the judgment. The net proceeds of any such resale must be credited to the buyer and payment of the judgment entitles him to any goods not resold.

 

(3)   After the buyer has wrongfully rejected or revoked acceptance of the goods or has failed to make a payment due or has repudiated (Section 2-610), a seller who is held not entitled to the price under this section shall nevertheless be awarded damages for non-acceptance under §2-708.

 

E. Incidental Damages and Consequential Damages

·         Generally, there are no consequential damages for the seller b/c they do not incur them when buyer breaches.

UCC § 2-710, Seller’s Incidental Damages and Consequential damages

(1)   Incidental damages to an aggrieved seller include any commercially reasonable charges or commissions incurred in stopping delivery, in the transportation, care, and custody of goods after the buyer’s breach, in connection w/ return or resale of the goods or otherwise resulting from the breach.

 

(2)   Consequential damages resulting from the buyer’s breach include any loss resulting from general or particular requirements and needs of which the buyer at the time of contracting had reason to know and which could not reasonably be prevented by resale or otherwise.

 

(3)   In a consumer K, seller may not recover consequential damages from a consumer.


 

TRANSFER OF GOODS

XX. Assignment and Delegation

            UCC § 2-210, Delegation of Performance; Assignment of Rights

(1)   If the seller or buyer assigns rights under a K, the following rules apply:

(a)   Either party may assign all their rights unless it would materially change the duty of the other party, increase materially the burden or risk imposed on that party by the K, or impair materially that party’s chance of obtaining return performance.  A right to damages for breach of the whole K or a right arising out of the assignor’s due performance of its entire obligation may be assigned despite agreement otherwise.

·         Ex: a singer cannot delegate his duty to sing to another singer b/c the duty to sing is personal, and the hiring party has a substantial interest in having his original singer perform.

·         The assignor is still liable for breach and must still make sure that his assignee fulfills his duty.

 

(b)   the creation, attachment, perfection, or enforcement of a security interest in the seller’s interest under a K is not an assignment that materially changes the duty of…see statute

(2)   If the buyer or seller delegates performance of its duties under a K, the following rules apply:

(a)   see statute

(b)   see statute

(c)    see statute

(d)   see statute

(3)   see statute

(4)   see statute


 

XXI. Derivative Title; Good Faith Purchasers; Entrustment

·         Voidable Title – a situation where a GF purchaser (Bona Fide purchaser, BFP) may purchaser greater title than the seller had. A GF purchaser (BFP) can prevail over the original owner if the GFP purchases the goods from someone who has voidable title.

 

UCC § 2-403, Power to Transfer; GF Purchase of Goods; “Entrusting”

(1)   A purchaser of goods acquires all title which his transferor had or had power to transfer except that a purchaser of a limited interest acquires rights only to the extent of the interest purchased (derivative title rule). A person w/ voidable title has power to transfer a good title to a good faith purchaser for value. When goods have been delivered under a transaction of purchase the purchaser has such power even though

(a)   The transferor was deceived as to the identity of the purchaser, OR

(b)   Delivery was in exchange for a check which is later dishonored, OR

(c)    It was agreed that the transaction was to be a “cash sale”, OR

(d)   The delivery was procured through fraud punishable as larcenous under the criminal law.

·         The above four subsections are ways to create voidable title.

·         Ex: If a thief steals a painting from someone’s home, and goes out and tries to sell it, then the buyer who purchases it from the thief will not prevail over the original seller b/c there is no way that the thief can get voidable title. Also, obviously, if the purchaser knew that the painting was stolen, then he wouldn’t be a BFP, so can’t prevail over original owner.

·         Ex: However, if the thief uses a more sophisticated scam, by telling the owner that he is an art dealer and convinces the owner that he has a prospective buyer of the painting, but the prospective buyer wants to inspect the painting first. The owner lets the thief leave w/ the painting. The thief then sells the painting to a BFP, who has no idea that the painting was stolen.

·         Since the owner entrusted the painting to the thief, then it can create voidable title and the BFP can prevailover the original owner. The original owner’s remedy will be to go after the thief. This is an exception to the derivative title rule.

·         However, the thief, if he couldn’t sell the painting, obviously could not prevail over the original owner.

(2)   Any entrusting of possession of goods to a merchant who deals in goods of that kind gives him power to transfer all rights of the entruster to a buyer in ordinary course of business.

(3)   “Entrusting” includes any delivery and any acquiescence in retention of possession regardless of any condition expressed between the parties…see the statute

·         Unless they are a merchant, they cannot be selling the goods in the ordinary course of business.

·         When you entrust the goods to a merchant that qualifies under subsection (2), you are giving him apparent authority.

 

Rank of Buyers, from Most protection to Least protection

1) Market Overt – this has never been recognized in the U.S. law. It is English law. Certain markets are market overt, where if you purchase something at a certain market, you have full title to it, no matter what, even if the goods were stolen, and you bought it from a thief. This is the only way a person can buy full title from a thief.

2) BIOC (Buyer in the ordinary course of business) – must be bona fide, must purchase from a merchant that sells goods of that kind, and the sale must be made in the ordinary course of business. 

3) BFP (Bona Fide purchaser)

3) Buyer (just a regular buyer)